PrepScholar

Choose Your Test

Sat / act prep online guides and tips, how to write a perfect synthesis essay for the ap language exam.

author image

Advanced Placement (AP)

body-pencil-sharpen-notebook-1

If you're planning to take the AP Language (or AP Lang) exam , you might already know that 55% of your overall exam score will be based on three essays. The first of the three essays you'll have to write on the AP Language exam is called the "synthesis essay." If you want to earn full points on this portion of the AP Lang Exam, you need to know what a synthesis essay is and what skills are assessed by the AP Lang synthesis essay.

In this article, we'll explain the different aspects of the AP Lang synthesis essay, including what skills you need to demonstrate in your synthesis essay response in order to achieve a good score. We'll also give you a full breakdown of a real AP Lang Synthesis Essay prompt, provide an analysis of an AP Lang synthesis essay example, and give you four tips for how to write a synthesis essay.

Let's get started by taking a closer look at how the AP Lang synthesis essay works!

Synthesis Essay AP Lang: What It Is and How It Works

The AP Lang synthesis essay is the first of three essays included in the Free Response section of the AP Lang exam.

The AP Lang synthesis essay portion of the Free Response section lasts for one hour total . This hour consists of a recommended 15 minute reading period and a 40 minute writing period. Keep in mind that these time allotments are merely recommendations, and that exam takers can parse out the allotted 60 minutes to complete the synthesis essay however they choose.

Now, here's what the structure of the AP Lang synthesis essay looks like. The exam presents six to seven sources that are organized around a specific topic (like alternative energy or eminent domain, which are both past synthesis exam topics).

Of these six to seven sources, at least two are visual , including at least one quantitative source (like a graph or pie chart, for example). The remaining four to five sources are print text-based, and each one contains approximately 500 words.

In addition to six to seven sources, the AP Lang exam provides a written prompt that consists of three paragraphs. The prompt will briefly explain the essay topic, then present a claim that students will respond to in an essay that synthesizes material from at least three of the sources provided.

Here's an example prompt provided by the College Board:

Directions : The following prompt is based on the accompanying six sources.

This question requires you to integrate a variety of sources into a coherent, well-written essay. Refer to the sources to support your position; avoid mere paraphrase or summary. Your argument should be central; the sources should support this argument .

Remember to attribute both direct and indirect citations.

Introduction

Television has been influential in United States presidential elections since the 1960's. But just what is this influence, and how has it affected who is elected? Has it made elections fairer and more accessible, or has it moved candidates from pursuing issues to pursuing image?

Read the following sources (including any introductory information) carefully. Then, in an essay that synthesizes at least three of the sources for support, take a position that defends, challenges, or qualifies the claim that television has had a positive impact on presidential elections.

Refer to the sources as Source A, Source B, etc.; titles are included for your convenience.

Source A (Campbell) Source B (Hart and Triece) Source C (Menand) Source D (Chart) Source E (Ranney) Source F (Koppel)

Like we mentioned earlier, this prompt gives you a topic — which it briefly explains — then asks you to take a position. In this case, you'll have to choose a stance on whether television has positively or negatively affected U.S. elections. You're also given six sources to evaluate and use in your response. Now that you have everything you need, now your job is to write an amazing synthesis essay.

But what does "synthesize" mean, exactly? According to the CollegeBoard, when an essay prompt asks you to synthesize, it means that you should "combine different perspectives from sources to form a support of a coherent position" in writing. In other words, a synthesis essay asks you to state your claim on a topic, then highlight the relationships between several sources that support your claim on that topic. Additionally, you'll need to cite specific evidence from your sources to prove your point.

The synthesis essay counts for six of the total points on the AP Lang exam . Students can receive 0-1 points for writing a thesis statement in the essay, 0-4 based on incorporation of evidence and commentary, and 0-1 points based on sophistication of thought and demonstrated complex understanding of the topic.

You'll be evaluated based on how effectively you do the following in your AP Lang synthesis essay:

Write a thesis that responds to the exam prompt with a defensible position

Provide specific evidence that to support all claims in your line of reasoning from at least three of the sources provided, and clearly and consistently explain how the evidence you include supports your line of reasoning

Demonstrate sophistication of thought by either crafting a thoughtful argument, situating the argument in a broader context, explaining the limitations of an argument

Make rhetorical choices that strengthen your argument and/or employ a vivid and persuasive style throughout your essay.

If your synthesis essay meets the criteria above, then there's a good chance you'll score well on this portion of the AP Lang exam!

If you're looking for even more information on scoring, the College Board has posted the AP Lang Free Response grading rubric on its website. ( You can find it here. ) We recommend taking a close look at it since it includes additional details about the synthesis essay scoring.

body-chisel-break-apart

Don't be intimidated...we're going to teach you how to break down even the hardest AP synthesis essay prompt.

Full Breakdown of a Real AP Lang Synthesis Essay Prompt

In this section, we'll teach you how to analyze and respond to a synthesis essay prompt in five easy steps, including suggested time frames for each step of the process.

Step 1: Analyze the Prompt

The very first thing to do when the clock starts running is read and analyze the prompt. To demonstrate how to do this, we'll look at the sample AP Lang synthesis essay prompt below. This prompt comes straight from the 2018 AP Lang exam:

Eminent domain is the power governments have to acquire property from private owners for public use. The rationale behind eminent domain is that governments have greater legal authority over lands within their dominion than do private owners. Eminent domain has been instituted in one way or another throughout the world for hundreds of years.

Carefully read the following six sources, including the introductory information for each source. Then synthesize material from at least three of the sources and incorporate it into a coherent, well-developed essay that defends, challenges, or qualifies the notion that eminent domain is productive and beneficial.

Your argument should be the focus of your essay. Use the sources to develop your argument and explain the reasoning for it. Avoid merely summarizing the sources. Indicate clearly which sources you are drawing from, whether through direct quotation, paraphrase, or summary. You may cite the sources as Source A, Source B, etc., or by using the descriptions in parentheses.

On first read, you might be nervous about how to answer this prompt...especially if you don't know what eminent domain is! But if you break the prompt down into chunks, you'll be able to figure out what the prompt is asking you to do in no time flat.

To get a full understanding of what this prompt wants you to do, you need to identify the most important details in this prompt, paragraph by paragraph. Here's what each paragraph is asking you to do:

  • Paragraph 1: The prompt presents and briefly explains the topic that you'll be writing your synthesis essay about. That topic is the concept of eminent domain.
  • Paragraph 2: The prompt presents a specific claim about the concept of eminent domain in this paragraph: Eminent domain is productive and beneficial. This paragraph instructs you to decide whether you want to defend, challenge, or qualify that claim in your synthesis essay , and use material from at least three of the sources provided in order to do so.
  • Paragraph 3: In the last paragraph of the prompt, the exam gives you clear instructions about how to approach writing your synthesis essay . First, make your argument the focus of the essay. Second, use material from at least three of the sources to develop and explain your argument. Third, provide commentary on the material you include, and provide proper citations when you incorporate quotations, paraphrases, or summaries from the sources provided.

So basically, you'll have to agree with, disagree with, or qualify the claim stated in the prompt, then use at least three sources substantiate your answer. Since you probably don't know much about eminent domain, you'll probably decide on your position after you read the provided sources.

To make good use of your time on the exam, you should spend around 2 minutes reading the prompt and making note of what it's asking you to do. That will leave you plenty of time to read the sources provided, which is the next step to writing a synthesis essay.

Step 2: Read the Sources Carefully

After you closely read the prompt and make note of the most important details, you need to read all of the sources provided. It's tempting to skip one or two sources to save time--but we recommend you don't do this. That's because you'll need a thorough understanding of the topic before you can accurately address the prompt!

For the sample exam prompt included above, there are six sources provided. We're not going to include all of the sources in this article, but you can view the six sources from this question on the 2018 AP Lang exam here . The sources include five print-text sources and one visual source, which is a cartoon.

As you read the sources, it's important to read quickly and carefully. Don't rush! Keep your pencil in hand to quickly mark important passages that you might want to use as evidence in your synthesis. While you're reading the sources and marking passages, you want to think about how the information you're reading influences your stance on the issue (in this case, eminent domain).

When you finish reading, take a few seconds to summarize, in a phrase or sentence, whether the source defends, challenges, or qualifies whether eminent domain is beneficial (which is the claim in the prompt) . Though it might not feel like you have time for this, it's important to give yourself these notes about each source so you know how you can use each one as evidence in your essay.

Here's what we mean: say you want to challenge the idea that eminent domain is useful. If you've jotted down notes about each source and what it's saying, it will be easier for you to pull the relevant information into your outline and your essay.

So how much time should you spend reading the provided sources? The AP Lang exam recommends taking 15 minutes to read the sources . If you spend around two of those minutes reading and breaking down the essay prompt, it makes sense to spend the remaining 13 minutes reading and annotating the sources.

If you finish reading and annotating early, you can always move on to drafting your synthesis essay. But make sure you're taking your time and reading carefully! It's better to use a little extra time reading and understanding the sources now so that you don't have to go back and re-read the sources later.

body-weightlifting-lift-strong

A strong thesis will do a lot of heavy lifting in your essay. (See what we did there?)

Step 3: Write a Strong Thesis Statement

After you've analyzed the prompt and thoroughly read the sources, the next thing you need to do in order to write a good synthesis essay is write a strong thesis statement .

The great news about writing a thesis statement for this synthesis essay is that you have all the tools you need to do it at your fingertips. All you have to do in order to write your thesis statement is decide what your stance is in relationship to the topic provided.

In the example prompt provided earlier, you're essentially given three choices for how to frame your thesis statement: you can either defend, challenge, or qualify a claim that's been provided by the prompt, that eminent domain is productive and beneficial . Here's what that means for each option:

If you choose to defend the claim, your job will be to prove that the claim is correct . In this case, you'll have to show that eminent domain is a good thing.

If you choose to challenge the claim, you'll argue that the claim is incorrect. In other words, you'll argue that eminent domain isn't productive or beneficial.

If you choose to qualify, that means you'll agree with part of the claim, but disagree with another part of the claim. For instance, you may argue that eminent domain can be a productive tool for governments, but it's not beneficial for property owners. Or maybe you argue that eminent domain is useful in certain circumstances, but not in others.

When you decide whether you want your synthesis essay to defend, challenge, or qualify that claim, you need to convey that stance clearly in your thesis statement. You want to avoid simply restating the claim provided in the prompt, summarizing the issue without making a coherent claim, or writing a thesis that doesn't respond to the prompt.

Here's an example of a thesis statement that received full points on the eminent domain synthesis essay:

Although eminent domain can be misused to benefit private interests at the expense of citizens, it is a vital tool of any government that intends to have any influence on the land it governs beyond that of written law.

This thesis statement received full points because it states a defensible position and establishes a line of reasoning on the issue of eminent domain. It states the author's position (that some parts of eminent domain are good, but others are bad), then goes on to explain why the author thinks that (it's good because it allows the government to do its job, but it's bad because the government can misuse its power.)

Because this example thesis statement states a defensible position and establishes a line of reasoning, it can be elaborated upon in the body of the essay through sub-claims, supporting evidence, and commentary. And a solid argument is key to getting a six on your synthesis essay for AP Lang!

Looking for help studying for your AP exam? Our one-on-one online AP tutoring services can help you prepare for your AP exams. Get matched with a top tutor who got a high score on the exam you're studying for!

Step 4: Create a Bare-Bones Essay Outline

Once you've got your thesis statement drafted, you have the foundation you need to develop a bare bones outline for your synthesis essay. Developing an outline might seem like it's a waste of your precious time, but if you develop your outline well, it will actually save you time when you start writing your essay.

With that in mind, we recommend spending 5 to 10 minutes outlining your synthesis essay . If you use a bare-bones outline like the one below, labeling each piece of content that you need to include in your essay draft, you should be able to develop out the most important pieces of the synthesis before you even draft the actual essay.

To help you see how this can work on test day, we've created a sample outline for you. You can even memorize this outline to help you out on test day! In the outline below, you'll find places to fill in a thesis statement, body paragraph topic sentences, evidence from the sources provided, and commentary :

  • Present the context surrounding the essay topic in a couple of sentences (this is a good place to use what you learned about the major opinions or controversies about the topic from reading your sources).
  • Write a straightforward, clear, and concise thesis statement that presents your stance on the topic
  • Topic sentence presenting first supporting point or claim
  • Evidence #1
  • Commentary on Evidence #1
  • Evidence #2 (if needed)
  • Commentary on Evidence #2 (if needed)
  • Topic sentence presenting second supporting point or claim
  • Topic sentence presenting three supporting point or claim
  • Sums up the main line of reasoning that you developed and defended throughout the essay
  • Reiterates the thesis statement

Taking the time to develop these crucial pieces of the synthesis in a bare-bones outline will give you a map for your final essay. Once you have a map, writing the essay will be much easier.

Step 5: Draft Your Essay Response

The great thing about taking a few minutes to develop an outline is that you can develop it out into your essay draft. After you take about 5 to 10 minutes to outline your synthesis essay, you can use the remaining 30 to 35 minutes to draft your essay and review it.

Since you'll outline your essay before you start drafting, writing the essay should be pretty straightforward. You'll already know how many paragraphs you're going to write, what the topic of each paragraph will be, and what quotations, paraphrases, or summaries you're going to include in each paragraph from the sources provided. You'll just have to fill in one of the most important parts of your synthesis—your commentary.

Commentaries are your explanation of why your evidence supports the argument you've outlined in your thesis. Your commentary is where you actually make your argument, which is why it's such a critical part of your synthesis essay.

When thinking about what to say in your commentary, remember one thing the AP Lang synthesis essay prompt specifies: don't just summarize the sources. Instead, as you provide commentary on the evidence you incorporate, you need to explain how that evidence supports or undermines your thesis statement . You should include commentary that offers a thoughtful or novel perspective on the evidence from your sources to develop your argument.

One very important thing to remember as you draft out your essay is to cite your sources. The AP Lang exam synthesis essay prompt indicates that you can use generic labels for the sources provided (e.g. "Source 1," "Source 2," "Source 3," etc.). The exam prompt will indicate which label corresponds with which source, so you'll need to make sure you pay attention and cite sources accurately. You can cite your sources in the sentence where you introduce a quote, summary, or paraphrase, or you can use a parenthetical citation. Citing your sources affects your score on the synthesis essay, so remembering to do this is important.

body-green-arrow-down

Keep reading for a real-life example of a great AP synthesis essay response!

Real-Life AP Synthesis Essay Example and Analysis

If you're still wondering how to write a synthesis essay, examples of real essays from past AP Lang exams can make things clearer. These real-life student AP synthesis essay responses can be great for helping you understand how to write a synthesis essay that will knock the graders' socks off .

While there are multiple essay examples online, we've chosen one to take a closer look at. We're going to give you a brief analysis of one of these example student synthesis essays from the 2019 AP Lang Exam below!

Example Synthesis Essay AP Lang Response

To get started, let's look at the official prompt for the 2019 synthesis essay:

In response to our society's increasing demand for energy, large-scale wind power has drawn attention from governments and consumers as a potential alternative to traditional materials that fuel our power grids, such as coal, oil, natural gas, water, or even newer sources such as nuclear or solar power. Yet the establishment of large-scale, commercial-grade wind farms is often the subject of controversy for a variety of reasons.

Carefully read the six sources, found on the AP English Language and Composition 2019 Exam (Question 1), including the introductory information for each source. Write an essay that synthesizes material from at least three of the sources and develops your position on the most important factors that an individual or agency should consider when deciding whether to establish a wind farm.

Source A (photo) Source B (Layton) Source C (Seltenrich) Source D (Brown) Source E (Rule) Source F (Molla)

In your response you should do the following:

  • Respond to the prompt with a thesis presents a defensible position.
  • Select and use evidence from at least 3 of the provided sources to support your line of reasoning. Indicate clearly the sources used through direct quotation, paraphrase, or summary. Sources may be cited as Source A, Source B, etc., or by using the description in parentheses.
  • Explain how the evidence supports your line of reasoning.
  • Use appropriate grammar and punctuation in communicating your argument.

Now that you know exactly what the prompt asked students to do on the 2019 AP Lang synthesis essay, here's an AP Lang synthesis essay example, written by a real student on the AP Lang exam in 2019:

[1] The situation has been known for years, and still very little is being done: alternative power is the only way to reliably power the changing world. The draw of power coming from industry and private life is overwhelming current sources of non-renewable power, and with dwindling supplies of fossil fuels, it is merely a matter of time before coal and gas fuel plants are no longer in operation. So one viable alternative is wind power. But as with all things, there are pros and cons. The main factors for power companies to consider when building wind farms are environmental boon, aesthetic, and economic factors.

[2] The environmental benefits of using wind power are well-known and proven. Wind power is, as qualified by Source B, undeniably clean and renewable. From their production requiring very little in the way of dangerous materials to their lack of fuel, besides that which occurs naturally, wind power is by far one of the least environmentally impactful sources of power available. In addition, wind power by way of gearbox and advanced blade materials, has the highest percentage of energy retention. According to Source F, wind power retains 1,164% of the energy put into the system – meaning that it increases the energy converted from fuel (wind) to electricity 10 times! No other method of electricity production is even half that efficient. The efficiency and clean nature of wind power are important to consider, especially because they contribute back to power companies economically.

[3] Economically, wind power is both a boon and a bone to electric companies and other users. For consumers, wind power is very cheap, leading to lower bills than from any other source. Consumers also get an indirect reimbursement by way of taxes (Source D). In one Texan town, McCamey, tax revenue increased 30% from a wind farm being erected in the town. This helps to finance improvements to the town. But, there is no doubt that wind power is also hurting the power companies. Although, as renewable power goes, wind is incredibly cheap, it is still significantly more expensive than fossil fuels. So, while it is helping to cut down on emissions, it costs electric companies more than traditional fossil fuel plants. While the general economic trend is positive, there are some setbacks which must be overcome before wind power can take over as truly more effective than fossil fuels.

[4] Aesthetics may be the greatest setback for power companies. Although there may be significant economic and environmental benefit to wind power, people will always fight to preserve pure, unspoiled land. Unfortunately, not much can be done to improve the visual aesthetics of the turbines. White paint is the most common choice because it "[is] associated with cleanliness." (Source E). But, this can make it stand out like a sore thumb, and make the gargantuan machines seem more out of place. The site can also not be altered because it affects generating capacity. Sound is almost worse of a concern because it interrupts personal productivity by interrupting people's sleep patterns. One thing for power companies to consider is working with turbine manufacturing to make the machines less aesthetically impactful, so as to garner greater public support.

[5] As with most things, wind power has no easy answer. It is the responsibility of the companies building them to weigh the benefits and the consequences. But, by balancing economics, efficiency, and aesthetics, power companies can create a solution which balances human impact with environmental preservation.

And that's an entire AP Lang synthesis essay example, written in response to a real AP Lang exam prompt! It's important to remember AP Lang exam synthesis essay prompts are always similarly structured and worded, and students often respond in around the same number of paragraphs as what you see in the example essay response above.

Next, let's analyze this example essay and talk about what it does effectively, where it could be improved upon, and what score past exam scorers awarded it.

To get started on an analysis of the sample synthesis essay, let's look at the scoring commentary provided by the College Board:

  • For development of thesis, the essay received 1 out of 1 possible points
  • For evidence and commentary, the essay received 4 out of 4 possible points
  • For sophistication of thought, the essay received 0 out of 1 possible points.

This means that the final score for this example essay was a 5 out of 6 possible points . Let's look more closely at the content of the example essay to figure out why it received this score breakdown.

Thesis Development

The thesis statement is one of the three main categories that is taken into consideration when you're awarded points on this portion of the exam. This sample essay received 1 out of 1 total points.

Now, here's why: the thesis statement clearly and concisely conveys a position on the topic presented in the prompt--alternative energy and wind power--and defines the most important factors that power companies should consider when deciding whether to establish a wind farm.

Evidence and Commentary

The second key category taken into consideration when synthesis exams are evaluated is incorporation of evidence and commentary. This sample received 4 out of 4 possible points for this portion of the synthesis essay. At bare minimum, this sample essay meets the requirement mentioned in the prompt that the writer incorporate evidence from at least three of the sources provided.

On top of that, the writer does a good job of connecting the incorporated evidence back to the claim made in the thesis statement through effective commentary. The commentary in this sample essay is effective because it goes beyond just summarizing what the provided sources say. Instead, it explains and analyzes the evidence presented in the selected sources and connects them back to supporting points the writer makes in each body paragraph.

Finally, the writer of the essay also received points for evidence and commentary because the writer developed and supported a consistent line of reasoning throughout the essay . This line of reasoning is summed up in the fourth paragraph in the following sentence: "One thing for power companies to consider is working with turbine manufacturing to make the machines less aesthetically impactful, so as to garner greater public support."

Because the writer did a good job consistently developing their argument and incorporating evidence, they received full marks in this category. So far, so good!

Sophistication of Thought

Now, we know that this essay received a score of 5 out of 6 total points, and the place where the writer lost a point was on the basis of sophistication of thought, for which the writer received 0 out of 1 points. That's because this sample essay makes several generalizations and vague claims where it could have instead made specific claims that support a more balanced argument.

For example, in the following sentence from the 5th paragraph of the sample essay, the writer misses the opportunity to state specific possibilities that power companies should consider for wind energy . Instead, the writer is ambiguous and non-committal, saying, "As with most things, wind power has no easy answer. It is the responsibility of the companies building them to weigh the benefits and consequences."

If the writer of this essay was interested in trying to get that 6th point on the synthesis essay response, they could consider making more specific claims. For instance, they could state the specific benefits and consequences power companies should consider when deciding whether to establish a wind farm. These could include things like environmental impacts, economic impacts, or even population density!

Despite losing one point in the last category, this example synthesis essay is a strong one. It's well-developed, thoughtfully written, and advances an argument on the exam topic using evidence and support throughout.

body-number-four-post-it-note

4 Tips for How to Write a Synthesis Essay

AP Lang is a timed exam, so you have to pick and choose what you want to focus on in the limited time you're given to write the synthesis essay. Keep reading to get our expert advice on what you should focus on during your exam.

Tip 1: Read the Prompt First

It may sound obvious, but when you're pressed for time, it's easy to get flustered. Just remember: when it comes time to write the synthesis essay, read the prompt first !

Why is it so important to read the prompt before you read the sources? Because when you're aware of what kind of question you're trying to answer, you'll be able to read the sources more strategically. The prompt will help give you a sense of what claims, points, facts, or opinions to be looking for as you read the sources.

Reading the sources without having read the prompt first is kind of like trying to drive while wearing a blindfold: you can probably do it, but it's likely not going to end well!

Tip 2: Make Notes While You Read

During the 15-minute reading period at the beginning of the synthesis essay, you'll be reading through the sources as quickly as you can. After all, you're probably anxious to start writing!

While it's definitely important to make good use of your time, it's also important to read closely enough that you understand your sources. Careful reading will allow you to identify parts of the sources that will help you support your thesis statement in your essay, too.

As you read the sources, consider marking helpful passages with a star or check mark in the margins of the exam so you know which parts of the text to quickly re-read as you form your synthesis essay. You might also consider summing up the key points or position of each source in a sentence or a few words when you finish reading each source during the reading period. Doing so will help you know where each source stands on the topic given and help you pick the three (or more!) that will bolster your synthesis argument.

Tip 3: Start With the Thesis Statement

If you don't start your synthesis essay with a strong thesis statement, it's going to be tough to write an effective synthesis essay. As soon as you finish reading and annotating the provided sources, the thing you want to do next is write a strong thesis statement.

According to the CollegeBoard grading guidelines for the AP Lang synthesis essay, a strong thesis statement will respond to the prompt— not restate or rephrase the prompt. A good thesis will take a clear, defensible position on the topic presented in the prompt and the sources.

In other words, to write a solid thesis statement to guide the rest of your synthesis essay, you need to think about your position on the topic at hand and then make a claim about the topic based on your position. This position will either be defending, challenging, or qualifying the claim made in the essay's prompt.

The defensible position that you establish in your thesis statement will guide your argument in the rest of the essay, so it's important to do this first. Once you have a strong thesis statement, you can begin outlining your essay.

Tip 4: Focus on Your Commentary

Writing thoughtful, original commentary that explains your argument and your sources is important. In fact, doing this well will earn you four points (out of a total of six)!

AP Lang provides six to seven sources for you on the exam, and you'll be expected to incorporate quotations, paraphrases, or summaries from at least three of those sources into your synthesis essay and interpret that evidence for the reader.

While incorporating evidence is very important, in order to get the extra point for "sophistication of thought" on the synthesis essay, it's important to spend more time thinking about your commentary on the evidence you choose to incorporate. The commentary is your chance to show original thinking, strong rhetorical skills, and clearly explain how the evidence you've included supports the stance you laid out in your thesis statement.

To earn the 6th possible point on the synthesis essay, make sure your commentary demonstrates a nuanced understanding of the source material, explains this nuanced understanding, and places the evidence incorporated from the sources in conversation with each other. To do this, make sure you're avoiding vague language. Be specific when you can, and always tie your commentary back to your thesis!

body-person-arrows-next

What's Next?

There's a lot more to the AP Language exam than just the synthesis essay. Be sure to check out our expert guide to the entire exam , then learn more about the tricky multiple choice section .

Is the AP Lang exam hard...or is it easy? See how it stacks up to other AP tests on our list of the hardest AP exams .

Did you know there are technically two English AP exams? You can learn more about the second English AP test, the AP Literature exam, in this article . And if you're confused about whether you should take the AP Lang or AP Lit test , we can help you make that decision, too.

Want to improve your SAT score by 160 points or your ACT score by 4 points?   We've written a guide for each test about the top 5 strategies you must be using to have a shot at improving your score. Download them for free now:

Ashley Sufflé Robinson has a Ph.D. in 19th Century English Literature. As a content writer for PrepScholar, Ashley is passionate about giving college-bound students the in-depth information they need to get into the school of their dreams.

Student and Parent Forum

Our new student and parent forum, at ExpertHub.PrepScholar.com , allow you to interact with your peers and the PrepScholar staff. See how other students and parents are navigating high school, college, and the college admissions process. Ask questions; get answers.

Join the Conversation

Ask a Question Below

Have any questions about this article or other topics? Ask below and we'll reply!

Improve With Our Famous Guides

  • For All Students

The 5 Strategies You Must Be Using to Improve 160+ SAT Points

How to Get a Perfect 1600, by a Perfect Scorer

Series: How to Get 800 on Each SAT Section:

Score 800 on SAT Math

Score 800 on SAT Reading

Score 800 on SAT Writing

Series: How to Get to 600 on Each SAT Section:

Score 600 on SAT Math

Score 600 on SAT Reading

Score 600 on SAT Writing

Free Complete Official SAT Practice Tests

What SAT Target Score Should You Be Aiming For?

15 Strategies to Improve Your SAT Essay

The 5 Strategies You Must Be Using to Improve 4+ ACT Points

How to Get a Perfect 36 ACT, by a Perfect Scorer

Series: How to Get 36 on Each ACT Section:

36 on ACT English

36 on ACT Math

36 on ACT Reading

36 on ACT Science

Series: How to Get to 24 on Each ACT Section:

24 on ACT English

24 on ACT Math

24 on ACT Reading

24 on ACT Science

What ACT target score should you be aiming for?

ACT Vocabulary You Must Know

ACT Writing: 15 Tips to Raise Your Essay Score

How to Get Into Harvard and the Ivy League

How to Get a Perfect 4.0 GPA

How to Write an Amazing College Essay

What Exactly Are Colleges Looking For?

Is the ACT easier than the SAT? A Comprehensive Guide

Should you retake your SAT or ACT?

When should you take the SAT or ACT?

Stay Informed

ap synthesis essay 2013

Get the latest articles and test prep tips!

Looking for Graduate School Test Prep?

Check out our top-rated graduate blogs here:

GRE Online Prep Blog

GMAT Online Prep Blog

TOEFL Online Prep Blog

Holly R. "I am absolutely overjoyed and cannot thank you enough for helping me!”

What are your chances of acceptance?

Calculate for all schools, your chance of acceptance.

Duke University

Your chancing factors

Extracurriculars.

ap synthesis essay 2013

How to Write the AP Lang Synthesis Essay + Example

Do you know how to improve your profile for college applications.

See how your profile ranks among thousands of other students using CollegeVine. Calculate your chances at your dream schools and learn what areas you need to improve right now — it only takes 3 minutes and it's 100% free.

Show me what areas I need to improve

What’s Covered:

What is the ap lang synthesis essay, how will ap scores affect my college chances.

AP English Language and Composition, commonly known as AP Lang, is one of the most engaging and popular AP classes offered at most high schools, with over 535,000 students taking the class . AP Lang tests your ability to analyze written pieces, synthesize information, write rhetorical essays, and create cohesive and concrete arguments. However, the class is rather challenging as only 62% of students were able to score a three or higher on the exam. 

The AP Lang exam has two sections. The first consists of 45 multiple choice questions which need to be completed in an hour. This portion counts for around 45% of your total score. These questions ask students to analyze written pieces and answer questions related to each respective passage.  All possible answer choices can be found within the text, and no prior knowledge of literature is needed to understand the passages.

The second section contains three free-response questions to be finished in under two hours and 15 minutes. This section counts for 55% of your score and includes the synthesis essay, the rhetorical essay, and the argumentative essay.

  • The synthesis essay requires you to read 6-7 sources and create an argument using at least three sources.
  • The rhetorical analysis essay requires you to describe how a piece of writing evokes specific meanings and symbolism.
  • The argumentative essay requires you to pick a perspective of a debate and create an argument based on the evidence provided.

In this post, we will take a look at the AP Lang synthesis essay and discuss tips and tricks to master this part of the exam. We will also provide an example of a well-written essay for review.  

The AP Lang synthesis essay is the first of three essays included in the Free Response section of the AP Lang exam. The exam presents 6-7 sources that are organized around a specific topic, with two of those sources purely visual, including a single quantitative source (like a graph or pie chart). The remaining 4-5 sources are text-based, containing around 500 words each. It’s recommended that students spend an hour on this essay—15 minute reading period, 40 minutes writing, and 5 minutes of spare time to check over work.

Each synthesis essay has a topic that all the sources will relate to. A prompt will explaining the topic and provide some background, although the topics are usually broad so you will probably know something related to the issue. It will also present a claim that students will respond to in an essay format using information from at least three of the provided sources. You will need to take a stance, either agreeing or disagreeing with the position provided in the claim. 

According to the CollegeBoard, they are looking for essays that “combine different perspectives from sources to form a support of a coherent position.” This means that you must state your claim on the topic and highlight relationships between several sources that support your specific position on the topic. Additionally, you’ll need to cite clear evidence from your sources to prove your point.

The synthesis essay counts for six points on the AP Lang exam. Students can receive 0-1 points for writing a thesis statement, 0-4 based on the incorporation of evidence and commentary, and 0-1 points based on the sophistication of thought and demonstration of complex understanding.

While this essay seems extremely overwhelming, considering there are a total of three free-response essays to complete, with proper time management and practiced skills, this essay is manageable and straightforward. In order to enhance the time management aspect of the test to the best of your ability, it is essential to divide the essay up into five key steps.

Step 1: Analyze the Prompt

As soon as the clock starts, carefully read and analyze what the prompt asks from you. It might be helpful to markup the text to identify the most critical details. You should only spend around 2 minutes reading the prompt so you have enough time to read all the sources and figure out your argument. Don’t feel like you need to immediately pick your stance on the claim right after reading the prompt. You should read the sources before you commit to your argument.

Step 2: Read the Sources Carefully

Although you are only required to use 3 of the 6-7 sources provides, make sure you read ALL of the sources. This will allow you to better understand the topic and make the most educated decision of which sources to use in your essay. Since there are a lot of sources to get through, you will need to read quickly and carefully.

Annotating will be your best friend during the reading period. Highlight and mark important concepts or lines from each passage that would be helpful in your essay. Your argument will probably begin forming in your head as you go through the passages, so you will save yourself a lot of time later on if you take a few seconds to write down notes in the margins. After you’ve finished reading a source, reflect on whether the source defends, challenges, or qualifies your argument.

You will have around 13 minutes to read through all the sources, but it’s very possible you will finish earlier if you are a fast reader. Take the leftover time to start developing your thesis and organizing your thoughts into an outline so you have more time to write. 

Step 3: Write a Strong Thesis Statement 

In order to write a good thesis statement, all you have to do is decide your stance on the claim provided in the prompt and give an overview of your evidence. You essentially have three choices on how to frame your thesis statement: You can defend, challenge or qualify a claim that’s been provided by the prompt. 

  • If you are defending the claim, your job will be to prove that the claim is correct .
  • If you are challenging the claim, your job will be to prove that the claim is incorrect .
  • If you choose to qualify the claim, your job will be to agree to a part of the claim and disagree with another part of the claim. 

A strong thesis statement will clearly state your stance without summarizing the issue or regurgitating the claim. The CollegeBoard is looking for a thesis statement that “states a defensible position and establishes a line of reasoning on the issue provided in the prompt.”

Step 4: Create a Minimal Essay Outline

Developing an outline might seem like a waste of time when you are up against the clock, but believe us, taking 5-10 minutes to outline your essay will be much more useful in the long run than jumping right into the essay.

Your outline should include your thesis statement and three main pieces of evidence that will constitute each body paragraph. Under each piece of evidence should be 2-3 details from the sources that you will use to back up your claim and some commentary on how that evidence proves your thesis.

Step 5: Write your Essay

Use the remaining 30-35 minutes to write your essay. This should be relatively easy if you took the time to mark up the sources and have a detailed outline.  Remember to add special consideration and emphasis to the commentary sections of the supporting arguments outlined in your thesis. These sentences are critical to the overall flow of the essay and where you will be explaining how the evidence supports or undermines the claim in the prompt.

Also, when referencing your sources, write the in-text citations as follows: “Source 1,” “Source 2,” “Source 3,” etc. Make sure to pay attention to which source is which in order to not incorrectly cite your sources. In-text citations will impact your score on the essay and are an integral part of the process.

After you finish writing, read through your essay for any grammatical errors or mistakes before you move onto the next essay.

Here are six must-have tips and tricks to get a good score on the synthesis essay:

  • Cite at least four sources , even though the minimum requirement is three. Remember not to plagiarize and cite everything you use in your arguments.
  • Make sure to develop a solid and clear thesis . Develop a stable stance for the claim and stick with it throughout the entire paper.
  • Don’t summarize the sources. The summary of the sources does not count as an argument. 
  • You don’t necessarily have to agree with the sources in order to cite them. Using a source to support a counterargument is still a good use of a source.
  • Cite the sources that you understand entirely . If you don’t, it could come back to bite you in the end. 
  • Use small quotes , do not quote entire paragraphs. Make sure the quote does not disrupt the flow or grammar of the sentence you write. 

ap synthesis essay 2013

Discover your chances at hundreds of schools

Our free chancing engine takes into account your history, background, test scores, and extracurricular activities to show you your real chances of admission—and how to improve them.

Here is an example prompt and essay from 2019 that received 5 of the 6 total points available:

In response to our society’s increasing demand for energy, large-scale wind power has drawn attention from governments and consumers as a potential alternative to traditional materials that fuel our power grids, such as coal, oil, natural gas, water, or even newer sources such as nuclear or solar power. Yet the establishment of large-scale, commercial-grade wind farms is often the subject of controversy for a variety of reasons.

Carefully read the six sources, found on the AP English Language and Composition 2019 Exam (Question 1), including the introductory information for each source. Write an essay that synthesizes material from at least three of the sources and develops your position on the most important factors that an individual or agency should consider when deciding whether to establish a wind farm.

Source A (photo)

Source B (Layton)

Source C (Seltenrich)

Source D (Brown)

Source E (Rule)

Source F (Molla)

In your response you should do the following:

  • Respond to the prompt with a thesis presents a defensible position.
  • Select and use evidence from at least 3 of the provided sources to support your line of reasoning. Indicate clearly the sources used through direct quotation, paraphrase, or summary. Sources may be cited as Source A, Source B, etc., or by using the description in parentheses.
  • Explain how the evidence supports your line of reasoning.
  • Use appropriate grammar and punctuation in communicating your argument.

[1] The situation has been known for years, and still very little is being done: alternative power is the only way to reliably power the changing world. The draw of power coming from industry and private life is overwhelming current sources of non-renewable power, and with dwindling supplies of fossil fuels, it is merely a matter of time before coal and gas fuel plants are no longer in operation. So one viable alternative is wind power. But as with all things, there are pros and cons. The main factors for power companies to consider when building wind farms are environmental boon, aesthetic, and economic factors.

[2] The environmental benefits of using wind power are well-known and proven. Wind power is, as qualified by Source B, undeniably clean and renewable. From their production requiring very little in the way of dangerous materials to their lack of fuel, besides that which occurs naturally, wind power is by far one of the least environmentally impactful sources of power available. In addition, wind power by way of gearbox and advanced blade materials, has the highest percentage of energy retention. According to Source F, wind power retains 1,164% of the energy put into the system – meaning that it increases the energy converted from fuel (wind) to electricity 10 times! No other method of electricity production is even half that efficient. The efficiency and clean nature of wind power are important to consider, especially because they contribute back to power companies economically.

[3] Economically, wind power is both a boon and a bone to electric companies and other users. For consumers, wind power is very cheap, leading to lower bills than from any other source. Consumers also get an indirect reimbursement by way of taxes (Source D). In one Texan town, McCamey, tax revenue increased 30% from a wind farm being erected in the town. This helps to finance improvements to the town. But, there is no doubt that wind power is also hurting the power companies. Although, as renewable power goes, wind is incredibly cheap, it is still significantly more expensive than fossil fuels. So, while it is helping to cut down on emissions, it costs electric companies more than traditional fossil fuel plants. While the general economic trend is positive, there are some setbacks which must be overcome before wind power can take over as truly more effective than fossil fuels.

[4] Aesthetics may be the greatest setback for power companies. Although there may be significant economic and environmental benefit to wind power, people will always fight to preserve pure, unspoiled land. Unfortunately, not much can be done to improve the visual aesthetics of the turbines. White paint is the most common choice because it “[is] associated with cleanliness.” (Source E). But, this can make it stand out like a sore thumb, and make the gargantuan machines seem more out of place. The site can also not be altered because it affects generating capacity. Sound is almost worse of a concern because it interrupts personal productivity by interrupting people’s sleep patterns. One thing for power companies to consider is working with turbine manufacturing to make the machines less aesthetically impactful, so as to garner greater public support.

[5] As with most things, wind power has no easy answer. It is the responsibility of the companies building them to weigh the benefits and the consequences. But, by balancing economics, efficiency, and aesthetics, power companies can create a solution which balances human impact with environmental preservation.

More examples can be found here at College Board.

While AP Scores help to boost your weighted GPA, or give you the option to get college credit, AP Scores don’t have a strong effect on your admissions chances . However, colleges can still see your self-reported scores, so you might not want to automatically send scores to colleges if they are lower than a 3. That being said, admissions officers care far more about your grade in an AP class than your score on the exam.

Related CollegeVine Blog Posts

ap synthesis essay 2013

How to Write the AP Lang Synthesis Essay with Example

September 5, 2023

AP Lang synthesis essay, AP Language

If you’re highly interested in learning more about writing analysis, then chances are you enrolled in AP Lang. Essentially, AP Lang is an advanced course for high schoolers that combines interest and knowledge in English with critical thinking. In the class, students learn how to analyze and synthesize a variety of texts to construct well-reasoned arguments. If you take AP Lang, then you can opt to take the AP test at the conclusion of the school year. On the exam, students write the AP Lang synthesis essay to demonstrate their learned abilities. In this article, we’ll look at what the AP Lang synthesis essay requires and show an example to provide better understanding of what to expect on the exam.

AP Lang Exam Basics

The AP Lang exam is separated into two sections. In the first section, students have one hour to answer a series of 45 multiple-choice questions. Here, about half of the questions are based on passages students read. The other half are focused on the best revision techniques. Essentially, the answers for the latter 20-22 questions are geared toward revising mock essays.

In this article, however, we’ll focus mainly on the second part of the exam: the AP Lang synthesis essay.

In this second section, students have two hours and 15 minutes to write three essays of their own design. The three open-ended questions in this section are intended to be free-response and allow for a variety of approaches. Each question is intended to allow up to 40 minutes to complete.

For the AP Lang synthesis essay, students are presented with a scenario of the College Board’s design. The scenario will provide its own thesis statement. Usually, scenarios relate to real-world problems like environmental concerns, media, or government policies.

For each scenario, students are provided with 6-7 outside sources. These sources could be in the form of an image, visual graph, or written paragraph. For written paragraphs, the sources are usually no more than 500 words.

Students are then expected to incorporate at least 3-4 of these outside sources into their essay response. The outside sources are intended to be used as supporting evidence for the student’s chosen stance or argument. Students are able to either agree with or disagree with the thesis presented in the original scenario.

AP Lang Exam – Scoring

In the second part of the AP Lang exam, students can earn a possible 6 points on each essay. 1 point is earned for the development of a thesis. Up to 4 points can be earned for evidence and commentary. The final 1 point is earned for sophistication of thought.

AP Lang Exam – Takeaways

Ultimately, the goal of the AP Lang synthesis essay is not whether the student is “right” or “wrong” in their argument. The key is that students are able to reasonably and clearly support their argument using the provided sources as evidence .

The College Board looks for your ability to identify relationships between texts , form a coherent argument , and interpret external sources .

Synthesis Essay AP Lang Examples

If you’re not sure how the questions will look on the AP Lang synthesis essay section, we’ll provide an example. After the example, we’ll break down the strengths and weaknesses of the response. That way, you’ll have a better idea of what the College Board is looking for.

Additionally, the College Board has released previous AP Lang synthesis essay examples you can review. They even have essay questions as recent as 2022 . For further support, a scoring commentary and comments from the Chief Reader are also available to view. Additionally, there are other examples you can view from earlier years .

Note: A good strategy to study for the synthesis essay AP Lang exam is to review your rhetorical devices and literary devices . Understanding how these devices function can be essential in constructing a cohesive essay.

Synthesis Essay AP Lang Examples – Sample Question

Below is a sample question from the AP Lang synthesis essay and a response to the prompt. This question was taken directly from a 2022 exam . However, the response to the question will be originally crafted for the purpose of this newsletter. As well, all supporting evidence will be originally created and does not correspond to any previous test.

The Question

Since the early 2000s, the United States government and a number of corporations have sponsored initiatives to improve education in the STEM disciplines: science, technology, engineering, and mathematics. The emphasis on STEM subjects in elementary, secondary, and higher education reflects concerns that United States students are less proficient in these areas than are students in other countries. Additionally, there is a belief that mastery in STEM fields is now essential in order to join a highly technical and specialized workforce. However, not everyone is convinced that a STEM-focused curriculum is necessary and/or effective.

In your response you should do the following:

  • Respond to the prompt with a thesis that presents a defensible
  • Select and use evidence from at least three of the provided sources to support your line of Indicate clearly the sources used through direct quotation, paraphrase, or summary. Sources may be cited as Source A, Source B, etc., or by using the description in parentheses.
  • Explain how the evidence supports your line of
  • Use appropriate grammar and punctuation in communicating your

How to Approach the Question

Maybe your first thought upon seeing this block of text is to feel overwhelmed. But don’t panic. There are effective ways to approach the question so you will be more prepared in your response.

It’s a good strategy to first isolate the thesis . What is the main idea of the text, and what is its argument?

Try it out. Reread the prompt and see if you can identify what the statement is asking you to develop an opinion on.

Think you’ve got it? In this example, we will be focusing on whether or not a STEM-focused curriculum in K-12 education is necessary and/or effective. In short, we will be arguing either for (highlighting the benefits) or against (highlighting the pitfalls) a STEM-focused curriculum.

How do we know what this statement is asking us?

Well, the statement provides a lot of background information. For example, we receive a definition of what STEM stands for. As well, we know that since 2000, there has been a greater initiative for STEM-focused classes.

When you read the prompt for the first time, it’s a great strategy to learn how to differentiate between background and contextual information from the heart of the argument .

A good way to learn how to isolate the argument is to look for transition words. Usually, these appear near the end of the question. Words like “however” and “yet” are signals that the statement is offering a differing opinion. Typically, the statement will tell you which two positions it’s offering for argument. These opinions are usually signaled by contrasting transition words.

So, now that we know what the question is asking us, what is the best way to respond?

Synthesis Essay AP Lang Examples – Sample Answer

The following is an essay response I crafted to the above question. After reading the sample, I will break down what it does well and what areas can be improved.

A STEM-focused curriculum is not as essential to providing a meaningful K-12 education. Because the majority of high school students are not proficient in STEM-focused classes, prioritizing these classes causes harm to student’s mental health and academic performance.

As seen in Source A, 60% of high school seniors in the Midwest only scored a C average in math and science-based classes (Langston). This statistic suggests that the majority of students do not resonate with STEM classes and therefore perform poorly. Earning a low score in any class does not bode well for students’ mental health.

When looking at the primary argument in Source C, it’s clear that most high schoolers prefer creative outlets to fact-based research (Kohler). Allowing students the opportunity to be more creative and initiate conversations about coursework lets students be more active in their learning. When students can discuss the nuance in their opinions, more personal growth happens. These conversations are not always easy to have in STEM-focused classes.

As well, when looking back to Source A, it’s clear that high school students in the Midwest earned higher grades, on average, in their English and art classes (Langston). This figure suggests that students perform better in these classes because they relate more to the source material. When relating to what they learn, they perform better in class.

In conclusion, STEM-focused curriculum is not as essential in K-12 education because most high school students do not relate to their STEM classes. When students do not earn satisfactory grades in these classes, it negatively affects their future college applications and job prospects.

Synthesis Essay AP Lang Examples – Answer Breakdown

So, what does this essay response get right, and where can it be improved? Let’s start with what the response does well.

First, the response establishes its thesis right away. Usually, it’s a good idea to clearly state your argument within the first paragraph. Not only is this a good practice because a reader can easily identify your stance, but also you can refer to your thesis as you write to make you stay on track.

With your thesis, it’s also a good idea to include one to two supporting sentences with the reasons why the thesis is concluded . Like in this example, I wrote that STEM-focused classes should not be prioritized because they can negatively affect both mental health and academic performance.

Another positive aspect of this response is that it is sure to not only reference but also cites its sources . It’s important that the reader understand where your information is coming from. That way, the readers can ensure you are interpreting the sources correctly.

AP Lang Synthesis Essay (Continued)

However, when rereading the instructions, it’s clear that this response fails the basic requirement of referring to at least three sources. Always make sure to reread the instructions to ensure you meet the standard requirements for incorporating source material.

Further, this AP Lang synthesis essay does not fully support its arguments . Ideas are simply stated and are not expanded upon.

For example, I mentioned a few times that earning low grades in STEM classes leads to negative mental health for high school students. However, there is no source referenced that either confirms or denies this claim. Therefore, there is no sufficient evidence to support my argument. It relies purely on inference.

Additionally, this AP Lang synthesis essay does not arrive at a sufficient level of sophistication of thought . Basically, sophistication of thought means avoiding broad generalizations and vague claims. The more specific you can be, the better your argument will sound.

Synthesis Essay AP Lang – In Conclusion

In the end, it’s always helpful to read the prompt thoroughly before writing. As well, making notes while you read could be a good strategy to pinpoint main ideas both in the prompt and the sources. That way, you can reread the material quickly. Similarly, sketching an outline may also be helpful. In addition, you should always carefully read the instructions to ensure all guidelines are followed.

As long as you avoid broad generalizations and use enough supporting evidence for your claim, you will be on the right path!

  • High School Success

' src=

Meghan Dairaghi

With a BA in English and an MFA in Creative Writing, Meghan has served as a writing tutor at the University of Missouri St. Louis and Maryville University. Additionally, Meghan has held editorial roles at River Styx and Boulevard, and was a prose reader at Farside Review . Most recently, her work has been featured in Belle Ombre , Flypaper Lit , and Mag 20/20 , among others, and she was nominated for the Mary Troy Prize in Fiction. 

  • 2-Year Colleges
  • Application Strategies
  • Best Colleges by Major
  • Best Colleges by State
  • Big Picture
  • Career & Personality Assessment
  • College Essay
  • College Search/Knowledge
  • College Success
  • Costs & Financial Aid
  • Dental School Admissions
  • Extracurricular Activities
  • Graduate School Admissions
  • High Schools
  • Law School Admissions
  • Medical School Admissions
  • Navigating the Admissions Process
  • Online Learning
  • Private High School Spotlight
  • Summer Program Spotlight
  • Summer Programs
  • Test Prep Provider Spotlight

College Transitions Sidebar Block Image

“Innovative and invaluable…use this book as your college lifeline.”

— Lynn O'Shaughnessy

Nationally Recognized College Expert

College Planning in Your Inbox

Join our information-packed monthly newsletter.

I am a... Student Student Parent Counselor Educator Other First Name Last Name Email Address Zip Code Area of Interest Business Computer Science Engineering Fine/Performing Arts Humanities Mathematics STEM Pre-Med Psychology Social Studies/Sciences Submit

pep

Find what you need to study

Synthesis Overview

10 min read • november 18, 2021

Justin Nazario

Justin Nazario

Attend a live cram event

Review all units live with expert teachers & students

Overview of the Synthesis Question

Section II of the AP English Language and Composition exam includes three free-response questions that you must answer in 2 hours and 15 minutes.

This guide will focus on Question 1 of Section II of the exam, the Synthesis question . As with all AP exams with free-response questions, the Synthesis question has its own rubric and scoring that we will detail later in this guide. 

To summarize, however, your essay should include/ demonstrate the following:

An easy to identify thesis 

Use of three or more of the provided sources

Explain how the sources used defend the claim in a complex manner

Writing that is sophisticated and collegiate

In the sections that follow, we will go over exactly what each part means. One thing to keep in mind is that the sources you choose should only strengthen your claim-- not step in and be the claim. Avoid overly citing from the sources to the point that your voice takes the backseat.

Luckily, the same skills of sophistication and complexity translate into the other essays you’ll write for this exam. Once you have developed your own voice, the rest is a matter of organization.

As stated before, you have 2 hours and 15 minutes to answer all three of your free-response questions. It seems like a lot, but it flies. To prevent getting behind schedule, it’s important to manage your time wisely.

A good breakdown to consider when pacing yourself is the following:

10 min. (to read sources) + 5 min. (planning) + 35 min. (writing) = 50 min.

How to Rock the Synthesis Question: The Rubric

The synthesis question is scored on a six-point rubric , and each point can be earned individually. This means that you can get points in one category, but not in others. It all depends on how well you accomplish each level on the rubric .

The Synthesis Question Rubric

Your thesis is the statement of your essay that introduces your claim to the reader. This is where you come forward and explicitly say: here is my position on the argument, and here are my reasons for feeling this way. 💭Above all else, you must respond to the prompt in its entirety. 

As in most essays, the introduction is recommended to be in the opening paragraph of your essay. ☝If it’s not in the introduction, you run the risk of confusing your reader, but your thesis can be anywhere in your essay. It can be as long as you’d like, so long as you present your main ideas in the order you will be discussing them in.

In order to receive the point, you need to both answer the prompt and present your own argument and claim to said prompt. A simple way to do so is to use words from the prompt to drive your thesis forward, but avoid just restating the thesis without adding your claim . You’ll lose out on the point if you forget to weave your argument into the thesis.

Your thesis and introductory paragraph are really where you introduce your style and voice as a writer. You have the opportunity to speak to your reader-- say something. Answer the prompt in complex, rich sentences that convey your use the sources to their highest potential. 👏

A great thesis does not have to be a paragraph long: as long as it answers the prompt, you’ll be alright!

Evidence and Commentary

This section on the rubric is split up into two categories: use of sources and commentary on the sources.

The College Board requires that you use at least three of the sources in order to earn the maximum amount of points. To “use” a source, you must cite text from the source or paraphrase an idea expressed by the author of the source, and then must explain its significance to the overall claim. (More on that in a moment.)

You must also establish a line of reasoning that the sources answer and/or incorporate into your elaboration. To make it a bit simpler, you need to explain how the source proves or challenges your claim. This can be accomplished in one sentence or several-- regardless, you need to explain why you chose to use that source to prove that claim. 

The second part of this category is the commentary section. Here, you must consistently establish the line of reasoning for each of the sources you introduce and do so with complexity. In all reality, this is just making sure that you are using each source for a reason, and not just fact-dropping information to earn the point. 

https://firebasestorage.googleapis.com/v0/b/fiveable-92889.appspot.com/o/images%2Fdownload-24.png?alt=media&token=22532fea-589c-4e86-aac9-8831dfe52dbb

An easy way to do this is by prefacing your citation with how the source relates to your argument, and then elaborating afterward.  Consider this example:

“The indoctrination of immigrants into American society is representative of a divide in American politics and culture, a line created by the two party system. (Source 2) Through the conditioning of immigrants to the ways of American society, there is a systematic erasing of native culture and ways in order to push American agendas onto people of other backgrounds and identities...”

The example drops the citation right in the middle of the paragraph in order to introduce the paraphrased idea, but divide it from the elaboration that follows:

Sophistication

The final row in the rubric is sophistication , or the level and complexity of your writing. This point is earned over the course of your essay and must be consistent in order for you to get the point.

This one is a little more complex to earn than some of the other points on the rubric . Contrary to the other rows, this is not something you need to directly set out to do, but something that needs to be developed over the course of your essay-- when you read a well-crafted sentence, you can tell. When you don’t read a well-crafted sentence, you can tell.

College Board has 4 notes on responses that typically earn this point:

Typically notice variations and conflicts within the sources , and explore said variations and conflicts

Express the restrictions of a source’s argument and does so within a larger scope and context

Demonstrate specific and powerful use of language so as to express professionalism and maturity

Use voice that is consistently lively yet coherent

Let’s break down each bullet.

The first bullet states is asking that your response acknowledges the difference between sources. Let’s say Source A is about how peanut butter is good for dogs but Source B says that peanut butter is actually harmful for dogs-- by expressing the counterpoints of the two sources, and discussing the broader context of the source and arguments presented in the two, you are demonstrating sophistication and can earn the point. The ‘explore’ part of the bullet is what makes or breaks it.

Make sure you don’t just drop things without explaining their significance or value!

The second bullet is relating the sources and information presented in them to both one another and the overall prompt. Ask yourself: What does this source talk about that this one doesn’t? How is the scope of this source relating to the prompt? What does this source say that this one builds off of? It’s about finding relationships between the sources and how, together, they make a set and rely on one another for validation or dejection. 👪

The third and fourth bullets are notes on your writing. The College Board wants to read essays and responses that are high quality and complex, not ones that lack development or are lackluster. They are really looking for responses that feel whole and complete, expressing entire thoughts rather than fragments of ideas that can get scattered and lost in translation. 

This mainly comes with practice and reading your peers’ work. Look for things such as sentence structure, diction, and punctuation. Do most of their sentences follow the same order and flow? Do they use the same three words to describe one thing or are they using a wide array of vocabulary? Think of how you can apply these things to your own writing, as well.

How to Rock the Synthesis Question - Process

Before you start writing....

Take time to plan your essays. If you just jump into writing without jotting down some ideas or a battle plan, you’re going to find yourself lost in the middle of your body paragraphs .

https://firebasestorage.googleapis.com/v0/b/fiveable-92889.appspot.com/o/images%2Fdownload-25.png?alt=media&token=cc8dafca-19cf-410e-814c-75084b98d8f1

A very simple idea for planning your essay is by using a template:

Main Idea #1

Supporting Detail #1

Evidence #1

Evidence #2

Elaboration (2-3 Sentences)

Supporting Detail #2

By organizing your ideas into an umbrella shape, you can get an idea of how your essay is going to read by the progression of your ideas. Remember that the order you present your ideas in must be the order you discuss them!

Another tip is to be 100% of what it is the prompt is asking of you. If the prompt is asking you to develop an argument or position on an event or idea, do exactly that. The sources tend to lend themselves towards one side of the argument, so be sure that whatever side you pick is well-supported with evidence from the sources. You can’t use any outside knowledge or anything that is not directly stated or implied by the sources. 

As mentioned before, it is extremely useful to use words in the prompt to formulate your thesis.

For example, if the prompt asks you what a country needs to consider before it engages in war with another country, you could formulate your thesis by saying “prior to engaging in war with another country, one must consider…” in order to directly respond to the question. This avoids confusion and allows you to easily pinpoint, for yourself, your thesis.

Think of all of Section 2 as a speech– this is the only section of the exam where you get to speak to the scorers. They are reading your handwriting, seeing your words and erase marks: make an impression! They are scored by a rubric , but they are also looking for voice and sophistication . Don’t brush off these essays and give minimal effort, they want you to pass.

Writing the Essay

Your introductory paragraph should realistically comprise of your thesis and introduce your response to the prompt. Your introduction can be just one sentence with your thesis, or you can build context by prefacing your argument or claim with things you learned from the sources. Avoid using “I”. 

Your body paragraphs should be where you spend most of your time writing. Remember what the rubric says about relationships and connections between the sources. Look for key similarities and differences that may lend you to choose a main idea from the set. They all have something in common!

After you have an idea of your main points, start with a topic sentence that is essentially a thesis for the paragraph. Explain what you’re going to discuss and how it relates back to the prompt (or broader context, if applicable).

After introducing your topic sentence , begin using your evidence and elaborating in complete, complex sentences. If you planned your essay well enough, you may even be able to just copy what you have written down and just spend time elaborating on the sources. This maximizes your time and gives you some space to develop an even more complex argument . 2-3 sentences of elaboration is the sweet spot if you cover all your bases.

After you’ve done the steps above, do the same for the next body paragraph.

Once you reach your conclusion , state for the final time your thesis and the points you mentioned in your body paragraphs . Someone should be able to read your conclusion and get a good idea of what it is you discussed in your response, so make it informative and a good representation of your work!

And once you’ve reached this point, you’re all done! Give your essay a read and fix any mechanical or grammatical issues that you may stumble upon. After that, move on to the next essay and keep your head high-- you’re one step closer to finishing the exam! ✋

Key Terms to Review ( 20 )

Body Paragraphs

Collegiate writing

Complex argument

Defensible position

Direct response to the prompt

Introductory paragraph

Line of Reasoning

Lively voice

Main Idea and Supporting Details

Planning your essay

Powerful use of language

Restrictions of a source's argument

Synthesis question

Time management

Topic Sentence

Variations and conflicts within the sources

Fiveable

Stay Connected

© 2024 Fiveable Inc. All rights reserved.

AP® and SAT® are trademarks registered by the College Board, which is not affiliated with, and does not endorse this website.

Calculate for all schools

Your chance of acceptance, your chancing factors, extracurriculars, ap synthesis essay examples.

Hey guys! I'm trying to get a better handle on the AP synthesis essay format and scoring. Do you know any reliable sources with solid examples and explanations? Thanks in advance!

Hi there! It's always a good idea to look for examples to better understand the format and expectations of an AP synthesis essay. I suggest you check out the following resources:

1. College Board: The College Board website is an excellent source as they create and administer the AP exams. You can find examples of synthesis essays from their past prompts, student responses, and scoring guidelines. Look for AP Language and Composition exams from previous years to find sample synthesis essay prompts and responses. Here's the link: https://apstudents.collegeboard.org/ap/2022-5

2. CollegeVine: CollegeVine has a helpful blog with numerous articles on AP exams, including tips on writing synthesis essays. While they may not have specific examples, their guidance on how to approach the synthesis essay can be very useful. Check out their website here: https://blog.collegevine.com/

3. AP Central: This is another reliable College Board resource that provides additional information about AP Language and Composition, including sample questions and responses. You can find it here: https://apcentral.collegeboard.org/

4. YouTube: Several teachers and educational channels on YouTube provide explanations and breakdowns of synthesis essay examples. One popular channel is Heimler's History, where he discusses the AP Language and Composition exam and provides tips and strategies for the synthesis essay. You can find his channel here: https://www.youtube.com/channel/UC_bOoi0e3xCAH6_mD1cN_AA

When reviewing examples, keep in mind the structure, organization, and style expected in a synthesis essay and pay attention to the integration of sources to support your position. Good luck with your practice!

About CollegeVine’s Expert FAQ

CollegeVine’s Q&A seeks to offer informed perspectives on commonly asked admissions questions. Every answer is refined and validated by our team of admissions experts to ensure it resonates with trusted knowledge in the field.

Mr Greg's English Cloud

Short Essay: AP Language Synthesis

The AP Language and Composition exam is a challenging test that requires students to demonstrate a wide range of skills. Among the various components of the exam, the synthesis essay is particularly demanding, as it necessitates not only strong writing abilities but also critical thinking and adept use of sources. In this article, we will delve into the synthesis essay, breaking down the steps and strategies necessary to craft a compelling response.

Table of Contents

Understanding the Synthesis Essay

At its core, the synthesis essay asks you to combine information from various sources to discuss a specific topic or argument. The College Board provides 6-7 sources that might include articles, essays, graphs, and images, reflecting different perspectives on an issue. Your task is to synthesize these sources into a coherent essay that presents your own position on the topic while incorporating the provided materials.

Step 1: Analyze the Prompt

Before you even consider the sources, read the prompt carefully. Understand exactly what it’s asking you to do. The prompt will typically present an issue and ask you to take a stance, using the sources to support your position. Identify keywords and consider any underlying assumptions or implications within the prompt.

Step 2: Examine the Sources

Take your time to read through each source provided. As you do, annotate key ideas, arguments, and pieces of evidence that might be useful. Be on the lookout for both supporting and contradicting points of view. Consider the credibility of each source and any biases that may be present.

Step 3: Develop a Thesis

Once you have a good grasp of the sources and the prompt, it’s time to formulate your thesis statement. Your thesis should clearly state your position on the issue and set the tone for your essay. It should be specific enough to guide your argument but flexible enough to allow for a nuanced discussion.

Step 4: Create an Outline

Organizing your thoughts and the information you plan to include is crucial. An outline will help you structure your essay logically. Plan out your introduction, thesis statement, body paragraphs, and conclusion. Decide where each source will fit into your argument and how you will address counterarguments.

Step 5: Write the Introduction

Your introduction should set the stage for your argument. Begin with a hook—an interesting fact, question, or quote that grabs the reader’s attention. Provide some background on the issue, if necessary, and then clearly state your thesis.

Step 6: Craft the Body Paragraphs

Each body paragraph should focus on a single idea that supports your thesis. Start with a topic sentence that clearly relates to your main argument. Integrate evidence from the sources, and explain how it supports your point. Be sure to attribute information correctly and blend the sources seamlessly into your own prose. Address counterarguments to demonstrate the depth of your analysis and reinforce your position.

Step 7: Use Transitions

Transitions are the glue that holds your essay together. They help the reader follow your argument from one point to the next. Use transitional phrases to connect ideas within paragraphs and to move smoothly between them.

Step 8: Conclude with Strength

Your conclusion should not simply restate your thesis. Instead, it should synthesize the main points you’ve made and demonstrate the broader implications of your argument. Leave your reader with a final thought that underscores the significance of your position.

Step 9: Cite Your Sources

While the AP Language exam does not require formal MLA or APA citation style, you do need to clearly indicate which source you are referencing. You might do this by mentioning the author’s name or by describing the source (“According to the graph provided…”).

Step 10: Revise and Proofread

If time permits, review your essay. Look for clarity, coherence, and grammatical accuracy. Make sure you’ve used a variety of sentence structures and that your word choice is precise. Confirm that you’ve accurately represented the sources and that your argument is persuasive.

Tips for Success

  • Understand Rhetorical Strategies : Knowing how to identify and discuss rhetorical devices will help you analyze the sources effectively.
  • Practice Critical Reading : The more you practice reading non-fiction texts critically, the better you will become at quickly identifying key arguments and evidence.
  • Practice Writing Under Time Constraints : The synthesis essay is timed, so you need to be able to organize and express your thoughts quickly and efficiently.
  • Learn to Synthesize : Synthesis goes beyond summarizing sources. It involves combining ideas from different sources to support your own argument in a cohesive way.
  • Expand Your Vocabulary : A rich vocabulary allows you to express your ideas clearly and persuasively.

Example Synthesis Prompt and Thesis

Prompt : In an increasingly digital world, libraries are at a crossroads. Some argue that libraries are more important than ever, providing access to digital resources and a public space for the community. Others believe that libraries are becoming obsolete in the age of the internet. Based on the sources provided, construct an argument about the role that libraries should play in contemporary society.

Thesis : In the 21st century, libraries must evolve beyond their traditional role as repositories of printed materials; they should serve as dynamic community centers that provide access to digital resources, promote digital literacy, and foster communal engagement.

With this thesis, the essay will not only synthesize the given sources but also present a nuanced perspective on the future of libraries.

AP Language Synthesis Essay Example #1

Prompt: Over the past decade, there has been a rising trend of telecommuting, with many individuals working from home rather than in traditional office settings. Some argue that telecommuting offers greater work-life balance and reduces the environmental impact of commuting. Others contend that it can lead to feelings of isolation and may hinder collaboration among colleagues. Based on the sources provided, write an essay that synthesizes at least three of the sources to develop your position on the pros and cons of telecommuting.

  • Source A: An article about the environmental benefits of telecommuting.
  • Source B: A study on productivity changes when employees work from home.
  • Source C: An interview with a psychologist discussing the social implications of telecommuting.
  • Source D: A business magazine article on the impact of telecommuting on company culture.
  • Source E: A blog post from a telecommuter sharing personal experiences.
  • Source F: An infographic with statistics on telecommuting trends.

Example Synthesis Essay

Title: Telecommuting: Balancing the Scale of Work-Life and Collaboration

In a world where the digital landscape is rapidly redefining the traditional workspace, telecommuting emerges as a beacon of modern employment. It promises a greener earth through fewer carbon emissions, a more balanced life away from the rigors of office commutes, and an autonomous work environment. However, these benefits come with trade-offs, such as potential obstacles to collaboration and a risk of worker isolation. By synthesizing insights from multiple sources, it becomes evident that telecommuting is a multifaceted issue that requires a nuanced approach to harness its benefits while mitigating its drawbacks.

The environmental argument for telecommuting, as presented in Source A, is compelling. The reduction in daily commutes translates to fewer cars on the road and a significant decrease in carbon footprint. This source posits that if employees with telework-compatible jobs worked from home even half the time, the national savings would total over $700 million in reduced oil consumption annually. The environmental benefits are clear, but they represent only one slice of the telecommuting pie.

Source B presents a more complex picture, suggesting that telecommuting can lead to increased productivity. The study details how employees often work longer hours from the comfort of their homes, free from the distractions of office politics and the time sink of daily commutes. However, this increased productivity does not necessarily equate to increased job satisfaction. The isolation mentioned in Source B can lead to a sense of disconnection from colleagues and the company culture.

The psychological impact of this isolation is further explored in Source C, where a psychologist asserts that human beings are inherently social creatures who thrive on interpersonal interactions. The psychologist warns that prolonged periods of isolation can lead to feelings of loneliness and can be detrimental to mental health—even more so in a work context where collaboration and team cohesion are essential.

Conversely, Source D argues that company culture does not have to suffer as a result of telecommuting. It proposes that with the right communication tools and regular in-person meetings, a company can maintain, if not strengthen, its culture. The article highlights companies that have successfully integrated telecommuting without sacrificing their collaborative ethos. This suggests that while telecommuting presents challenges to maintaining company culture, these challenges are not insurmountable with intentional effort.

The personal account in Source E provides a testament to the positive aspects of telecommuting, outlining the writer’s improved work-life balance, increased autonomy, and reduced stress levels. This perspective is valuable as it underscores the importance of individual differences. For some, the solitude of telecommuting is a blessing that allows for deep work and concentration, whereas for others, it can be a curse that breeds isolation and disconnection.

The infographic in Source F offers a broader overview, presenting data that reflects the growing trend of telecommuting. It states that telecommuting has increased by 44% over the last five years, suggesting that the workforce is adapting to this new model. This rise indicates that despite its challenges, telecommuting is meeting the needs of a significant portion of the workforce.

Considering the evidence, telecommuting presents a paradoxical challenge: it can both enhance and diminish the quality of the working experience. On the one hand, it can lead to environmental benefits, increased productivity, and improved work-life balance (Sources A, B, and E). On the other hand, it can impede collaboration and contribute to feelings of isolation (Sources C and D). The solution lies not in rejecting telecommuting outright but in approaching it with a strategy that fosters connection and collaboration while allowing for the flexibility that many workers desire.

In conclusion, telecommuting is not a one-size-fits-all solution. It is a flexible tool that, when used appropriately, can offer significant benefits to both the employee and the environment. Organizations and employees alike must strive to strike a balance—leveraging technology to maintain team unity, instituting regular check-ins to combat isolation, and promoting a culture that values both individual and collective success. As the workplace continues to evolve, so must our strategies for ensuring that telecommuting serves as a bridge to a more sustainable and satisfying professional life, rather than a barrier.

This synthesis essay integrates material from at least three of the provided sources to construct an argument. It carefully balances the pros and cons of telecommuting, providing a nuanced analysis of the issue. Remember that actual AP sources will contain more detailed information, and your essay should include direct reference to the content of the sources, including quotations and specific data where appropriate. Always cite the sources as Source A, Source B, etc., within your essay.

AP Language Synthesis Essay Example #2

Prompt: The rise of social media platforms has transformed the way people interact and consume information. While some praise social media for democratizing information and connecting people worldwide, others criticize it for contributing to the spread of misinformation and reducing the quality of public discourse. Based on the sources provided, write an essay that synthesizes at least three of the sources to argue whether social media has been more beneficial or detrimental to society.

  • Source A: An article on the role of social media in political movements.
  • Source B: A study examining the relationship between social media use and mental health.
  • Source C: A journalist’s opinion piece on the spread of misinformation through social media.
  • Source D: A report on the impact of social media on local businesses and economies.
  • Source E: A survey revealing how different generations use social media.
  • Source F: An interview with a social media platform developer discussing the intent behind the platform’s design.

Title: Navigating the Digital Maze: The Dual Faces of Social Media

In an age where a tweet can spark a revolution and a hashtag can unite millions, social media has become a fundamental pillar of modern communication. Its rapid ascendancy has ushered in a new era, where information is at the fingertips of anyone with internet access. This power to connect has been instrumental in driving political movements, as noted in Source A, which highlights the crucial role social media played in the Arab Spring. However, this connectivity comes at a price. As social media’s influence deepens, its capacity for harm seems to match its potential for good. The debate on whether social media serves as a force for societal benefit or detriment is multifaceted, and a synthesis of the provided sources reveals a complex web of consequences that social media weaves.

The democratizing power of social media is evident in its ability to mobilize and empower individuals, particularly in political contexts. Source A illustrates this through examples of social media as a tool for political activism, enabling citizens to organize, protest, and hold authorities accountable. This represents a significant shift in power dynamics, from a centralized broadcast model to a decentralized network where voices that were previously marginalized can now be amplified.

Despite these positive aspects, the argument that social media is a double-edged sword is well-founded. Source B presents alarming evidence linking excessive social media use to deteriorating mental health, especially among younger demographics. The constant comparison with others, cyberbullying, and the “fear of missing out” are cited as contributing factors to anxiety and depression. This correlation points to a darker side of social networks, where the quest for likes and follows can eclipse the quest for genuine human connection.

The pernicious spread of misinformation through social media platforms, as discussed in Source C, adds another layer of complexity. The piece elucidates how false information can go viral, sometimes with dire consequences. The recent proliferation of “fake news” has led to widespread confusion and a general mistrust in the information ecosystem. Social media, designed to facilitate the rapid exchange of information, has inadvertently become a conduit for deception on a grand scale.

However, not all effects of social media are so grim. Source D sheds light on its positive economic impacts, particularly for local businesses. By providing a platform for advertising and customer engagement, social media has enabled small businesses to flourish. The report suggests that social media has been a boon for entrepreneurship, offering affordable and effective marketing tools that were previously accessible only to large companies with substantial budgets.

The generational divide in social media use, as shown in Source E, suggests that the platform’s effects are not uniform across all age groups. While younger users may be more prone to the negative social impacts, older generations might leverage social media primarily for staying connected with family and friends, thus experiencing more of its benefits.

Lastly, Source F provides insight from the perspective of those behind the curtain: the developers of social media platforms. The interviewee reflects on the original intent of these networks—to connect people—and acknowledges that while the platforms have largely succeeded in this goal, they have also created unintended negative consequences.

In synthesizing these sources, it becomes clear that social media is neither wholly beneficial nor entirely detrimental. It is a reflection of humanity itself—capable of great good and great harm. The key lies in how individuals and societies choose to utilize this tool. Social media has the potential to be a formidable ally in the quest for a more informed and connected world, as long as users remain vigilant against its darker tendencies.

In conclusion, the impact of social media on society is akin to fire—a powerful element that can warm a home or raze it. It is incumbent upon users, platform developers, and policymakers alike to cultivate a virtual environment that fuels the flames of positive change while containing the destructive blazes of harm. As social media continues to evolve, so too should our understanding and regulation of its vast influence.

This essay integrates material from the provided sources and develops an argument about the complex nature of social media’s impact on society. It weaves together the benefits and drawbacks discussed in the sources to present a nuanced view that recognizes the power of social media as a tool whose ultimate effect depends on its use and regulation. Remember, in an actual AP Synthesis Essay, you would need to reference the sources more directly, including specific evidence and quotations, and provide in-text citations (Source A, Source B, etc.) within the body of your essay.

AP Language Synthesis Essay Example #3

Prompt: In recent years, the movement towards organic and locally sourced foods has gained momentum. Advocates claim that this shift is essential for health, environmental sustainability, and supporting local economies. Critics argue that it is an inefficient use of resources and can be economically divisive. Based on the sources provided, write an essay that synthesizes at least three of the sources to evaluate the validity of the movement towards organic and locally sourced foods.

  • Source A: A study on the health benefits of organic foods.
  • Source B: An economic analysis comparing the costs of local vs. imported foods.
  • Source C: An article on the environmental impact of local sourcing.
  • Source D: A food critic’s blog post questioning the superiority of organic food taste.
  • Source E: A sociologist’s research paper on food deserts and community access to organic foods.
  • Source F: A farmer’s interview about the challenges and rewards of local farming.

Title: The Organic Odyssey: Weighing the Costs and Benefits of a Local Plate

The push for organic and locally sourced foods has transformed from a niche interest into a significant cultural and economic movement. Proponents hail it as a panacea for a range of issues from health to the environment, while critics decry it as an impractical and elitist trend that exacerbates social inequalities. This dichotomy necessitates a critical examination of the widespread claims surrounding the movement. By drawing from various perspectives outlined in the provided sources, it is possible to construct a more measured appraisal of the push for organic and local food sourcing.

Source A presents compelling evidence on the health advantages of organic foods, citing lower pesticide levels and higher nutritional content. These health benefits provide a strong incentive for consumers to seek out organic options, potentially leading to better overall public health outcomes. Yet, the study does not fully address the economic implications of choosing organic, which can be significantly more expensive than conventionally grown produce.

The economic argument against the exclusive consumption of organic and local foods is expounded in Source B. The analysis reveals that local foods often come with a higher price tag due to the economies of scale enjoyed by larger, non-local farms. This cost difference can make organic and locally sourced foods less accessible to lower-income individuals, inadvertently contributing to a socioeconomic divide.

Environmental considerations are the focus of Source C, which underscores the reduced carbon footprint associated with local sourcing. By eliminating long-distance transportation, local sourcing minimizes greenhouse gas emissions, which is a clear environmental win. This source suggests that the benefits of local sourcing extend beyond the individual to global ecological health, reinforcing the environmental argument in favor of the local food movement.

However, Source D introduces a contrarian viewpoint, challenging the purported taste superiority of organic foods. The food critic argues that taste is subjective and that organic foods do not consistently outperform their non-organic counterparts in blind taste tests. This perspective is a reminder that personal preference should not be overlooked in the discourse on food sourcing and that taste alone may not justify the higher cost of organic foods for many consumers.

The issue of access is further complicated in Source E, which investigates food deserts—urban and rural areas where fresh, healthy food is difficult to obtain. The sociologist’s research highlights the disparity in access to organic foods, emphasizing that for many communities, the debate over organic versus conventional is moot when availability is the primary concern. This research implies that the movement toward organic and local foods must be accompanied by efforts to increase access and affordability if it is to be truly transformative.

Finally, Source F provides insight into the experiences of a local farmer, revealing the challenges faced by small-scale agriculture. The interview conveys the passion and dedication of local farmers and the value they place on sustainable practices. It also brings to light the financial and logistical hurdles that these farmers encounter in a market dominated by large agribusinesses.

Synthesizing these sources, it becomes evident that the movement toward organic and locally sourced foods is neither an unequivocal good nor an outright misstep. It offers tangible benefits in health and environmental sustainability but also poses significant economic and access challenges. The validity of this movement, therefore, hinges on a delicate balance: promoting the growth of organic and local food markets while ensuring that these options are affordable and accessible to all segments of the population.

In conclusion, the organic and local food movement has the potential to be a force for positive change, provided that it does not become an exclusive club for the affluent. Bridging the gap between healthful, sustainable food and the diverse needs of a broad consumer base requires policy innovations, community support initiatives, and continued dialogue. As the movement evolves, it must strive to include rather than divide, to nourish both the body and the fabric of society.

In this synthesis essay, the sources provided are integrated to explore the complexity of the organic and locally sourced food movement. The essay examines the movement from multiple angles, considering health, economic, environmental, taste preference, accessibility, andpersonal perspectives. It does not outright reject or accept the movement but rather calls for a nuanced approach that acknowledges its benefits and addresses its limitations. In an actual AP Synthesis Essay, direct references to the sources with in-text citations (Source A, Source B, etc.) would be included to support the points made, and quotations might be used for evidence. The essay would demonstrate how the writer can engage with multiple viewpoints and synthesize them into a coherent argument.

Final Thoughts

The synthesis essay on the AP Language exam presents a unique opportunity to showcase your ability to think critically, argue effectively, and integrate information from multiple sources. By understanding the prompt, examining the sources critically, developing a strong thesis, organizing your thoughts clearly, and employing rhetorical strategies skillfully, you can compose an essay that demonstrates your readiness for college-level work and beyond.

Remember, the key to success is practice. The more you work on reading critically, writing essays, and synthesizing information, the more adept you will become at this challenging but rewarding task. Good luck on your AP Language journey!

About Mr. Greg

Mr. Greg is an English teacher from Edinburgh, Scotland, currently based in Hong Kong. He has over 5 years teaching experience and recently completed his PGCE at the University of Essex Online. In 2013, he graduated from Edinburgh Napier University with a BEng(Hons) in Computing, with a focus on social media.

Mr. Greg’s English Cloud was created in 2020 during the pandemic, aiming to provide students and parents with resources to help facilitate their learning at home.

Whatsapp: +85259609792

[email protected]

ap synthesis essay 2013

logo-type-white

AP® English Language

The ultimate guide to 2013 ap® english language frqs.

  • The Albert Team
  • Last Updated On: March 1, 2022

the_ultimate_guide_to_2013 AP® English language frqs

AP® English Language FRQs

Imagine you are watching your favorite show one night. All of a sudden your television flashes and shuts off. Try as you might, you can’t get it to work. You decide that you’ll have to go down to the store and pick out a new one.

On the way to the store, you pass a man handing out small cards. The cards say that they are good for up to $1,000 off the purchase of a television from the store. Of course you would take the card and buy the television with it at an incredible discount.

This scenario is similar to the way that AP® exams work for college-bound students. Much like the store taking a large part of the cost of a television, colleges will offer credit for students that pass an AP® exam. What students don’t often realize is that passing just one AP® exam can save you thousands of dollars of tuition costs.

It seems obvious, but you should take the exam seriously. The offer has been made, but it is up to you to accept it. If you do accept, it means you will study harder and prepare yourself so that you can pass. This guide exists to help you seize the opportunity.

In this guide we have compiled the do’s and don’ts of the 2013 AP® English Language FRQ section, to provide you with the best information to conquer the exam. As you prepare for the exam, keep a close watch for the best practices for each type of essay, and the things to avoid in your writing.

Let’s break down the test to see how it is scored and what you’re expected to do.

Test Breakdown

The Free Response Questions (FRQs) are the essay portion of the AP® Language exam. The exam itself has two parts; the first is a multiple-choice section, and the second is the FRQs. This guide provides an overview, some strategies, and some examples of the FRQs from the CollegeBoard. There is a guide to the multiple choice here .

The FRQ section has two distinct parts: 15 minutes for reading a set of texts and 120 minutes for writing three essays. The 15 minute “reading period” is designed to give you time to read through the documents for question one and develop a thoughtful response. Although you are advised to give each essay 40 minutes, there is no set amount of time for any of the essays. You may divide the 120 minutes however you want.

The three FRQs are each designed to test a different style of writing. The first question is always a synthesis essay – which is why they give you 15 minutes to read all of the sources you must synthesize. The second essay is rhetorical analysis, requiring you to analyze a text through your essay. The third paper is an argumentative essay.

Each essay is worth one-third of the total grade for the FRQ section, and the FRQ section is worth 55% of the total AP® test. Keep that in mind as you prepare for the exam that, while the multiple-choice section is hard, the essays are worth more overall – so divide your study time evenly.

The scale for essay scores ranges from 1-9. A score of 1 suggests your essay is illegible or unintelligible, while a score of 9 is going to reflect the best attributes and aspects of early college-level writing. You should be shooting to improve your scores to the passing range, which is 5 or above. Note that if you are struggling with the multiple choice section, a 9-9-9 on the essays can help make up for it.

The Tale of Three Essays

If you are currently taking an AP® class, you have probably experienced the style and formats of the three assignments. You may have learned about the specifics of the different types of essays in class, and you may have already found out which of the three is easiest for you. However, you must possess skill in all three to master the AP® test.

The First Essay (Synthesis)

The first essay on the test is the synthesis essay . This essay can be the trickiest to master, but once you do get the hang of it, you will be one step closer to learning the others. The synthesis requires you to read seven texts, which can be poems, articles, short stories, or even political cartoons.

Once you have read and analyzed the texts, you are asked to craft an argument using at least three of the documents from the set. The sources should be used to build and support your argument, and you must integrate them into a coherent whole.

On the 2013 FRQ section of the AP® exam, the synthesis essay focuses on the planning and consideration that goes into building a monument. The complete prompt for the section is below:

AP® English Language FRQs

If we break down the task, it is asking you to use the six sources to create a “coherent, well-developed argument” from your position about the factors a group or agency should consider when memorializing a person in a monument. As you read this, you might have some experience with the topic or have seen monuments in your life. You can use that experience, but your response needs to focus on the given texts.

To find the actual documents you can go here . Taking a look at the documents will provide some context for the essay samples and their scores.

The question is scored on a scale from 1-9, with nine being the highest. Let’s take a look at some examples of student essays, along with comments from the readers – to break down the dos and don’ts of the FRQ section.

You should always strive to get the highest score possible. Writing a high-scoring paper involves learning some practices that will help you write the best possible synthesis essay. Below are two examples taken from student essays.

Create a Clear Thesis

One of the key elements of scoring high on the synthesis essay is to make your argument as clear as possible. Let’s look at the clarity in the example below:

AP® English Language FRQs

This sample comes from a high scoring essay. In particular, this student makes her argument clear through the thesis she crafts. The language that the student uses makes it clear what she is talking about, “careful consideration of its location, size, material, and purpose can effectively pay homage to the deep sacrifice or honor moments of great achievement”.

In her thesis statement, the student points out the four reasons she will use to make her argument: location, size, material, and purpose. She also drives home the importance of those topics in connection with the overall use of monuments, to “pay homage”.

For the reader of this essay, the student wrote very clear what she will discuss in the essay. The clarity in the thesis makes this essay much easier to read and understand, which will result in more points.

Explain Sources in Detail

Another essential part of scoring well on the synthesis essay is to explain your sources in great detail. The student example below demonstrates the skill:

AP® English Language FRQs

The student who wrote this essay was able to explain in great detail how the evidence works to support her argument. She can use her writing to examine how location works for or against the value of a monument by illustrating the difference for the reader.

In the case of this paragraph, she shows how a statue of Columbus fits in a park, a lovely scene where it belongs, and how it doesn’t fit with the scenery of an abandoned building or by billboards. The images she creates help to demonstrate her point well.

As you determine which sources to use as evidence, you will want to work on how you will explain or connect the evidence to your main point. You need to have valid reasoning that uses the evidence to drive home the argument.

There are some practices that students should avoid on FRQ 1 of the test. Students who do these things can expect to receive low scores on their essays, and if you wish to score above a five, you should avoid them at all costs.

Don’t Fail to Address the Prompt

One of the biggest mistakes students make is going off topic with the essay. Many students neglect the prompt and instead write around the issue, which results in a very low score. Let’s look at a low-scoring example of writing around the prompt:

AP® English Language FRQs

This student doesn’t seem to understand the main point of the essay. Instead of explaining what needs consideration when planning a monument, the student goes off on a tangent about how the “government does not need to spend an excessive amount of money on memorials…”

The topic of monument cost could have fit into the prompt, but making it the main issue doesn’t set this essay up to succeed. Instead of addressing the important considerations before erecting a monument, this student has challenged the entire idea of monuments.

In your essay, make sure that you always work to address the prompt. Your score will depend on you arguing the issue presented in the prompt.

Don’t Use Assumptions to Argue

When writing an argumentative essay, it is good practice to focus on facts and evidence and not jump to conclusions. In the example below the student works off of an assumption that isn’t substantiated by the evidence given:

AP® English Language FRQs

This student makes the assumption early on that “the Lincoln Memorial has no importance to some”. While the evidence used does demonstrate that the memorial can be discounted as “memorial,” it doesn’t show that people don’t think it is important.

The student needs to work towards an understanding that can be proven by the evidence he uses. The lack of proof to demonstrate their point makes their argument weak and accounts for the loss of points.

Always make sure that you can support your reasons and argument with substantial evidence. If the evidence you use doesn’t support what you are saying, you may want to change your argument to be underpinned by the sources.

AP® Readers’ Tips:

  • Read every text before you start your essay. A common pitfall is that many students do not use enough sources and try to fit them in after the fact.
  • Plan ahead. Ensure that you understand what you are going to be saying and how you will incorporate the sources into your writing. You will need at least three sources to get above a 6, so ensure you have at least that many mapped in your plan.

The Second Essay (Rhetorical Analysis)

The second essay on the FRQ section is always a rhetorical analysis essay. This essay will focus on analyzing a text for an important aspect of the writing. In the case of the 2013 FRQ, the analysis was supposed to concentrate on rhetorical strategies:

AP® English Language FRQs

The prompt asks the reader to carefully read a chapter from Richard Louv’s Last Child in the Woods and analyze the strategies that he uses to create and argument about the separation of people and nature. Rhetorical strategies include things like the rhetorical appeals and rhetorical devices.

Let’s examine the do’s and don’ts for the second essay.

When analyzing rhetorical strategies, you should pay close attention to the details within the text. The students below use some valuable strategies to enhance their analysis.

Introduce the Topic with Background Knowledge

Background knowledge is one of the most efficient tools to use in the second essay. Students that have background knowledge tend to write much more detailed and meaningful essays. Let’s look at an example below:

AP® English Language FRQs

In the example above, the student uses her knowledge of ancient Egypt to help explain her argument. The prompt asks students to examine Louv’s argument about humanity’s connection to nature – and the student introduces the topic by explaining how fundamental nature has been to humanity’s development.

The student can use her background knowledge to show that nature was intimately connected to all facets of life in the past. She goes on to explore how that relationship has changed in the present, but the examination helps draw out the comparison and sets her up to explain Louv’s argument.

If you can utilize background knowledge on a topic or argument, you should do it. The use of that knowledge will help shape your essay and can add depth that wouldn’t be accessible otherwise.

Use Direct Evidence from the Text

Using direct evidence from the text works to enhance your writing. Without direct evidence, your reasoning would have nothing to support it. Let’s take a look at how a student did this on the 2013 exam:

AP® English Language FRQs

In the example above, the student goes into great detail about what Louv says in his story. However, the student also manages to pull direct quotes from the text to support what she is explaining from the text.

The use of direct quotations serves the student well, and she is ultimately able to show exactly what Louv was saying through her use of text. The example helps illustrate her point and supports her argument.

If you utilize direct quotations well it can go a long way towards earning you a nine on the essay. You must learn how to integrate the quotes seamlessly, and how to use them effectively with your reasoning.

Some things to avoid on the literary analysis essay include providing little evidence and adding fluff.

Don’t Write an Argument without Providing Direct Evidence

One way to miss crucial points is to neglect direct evidence in your argument. Just as adding direct evidence makes your argument stronger, choosing to argue without it will make it look weak. Let’s look at one of the examples of this from a student essay:

AP® English Language FRQs

The student discusses the strategy of using direct quotations but doesn’t offer any examples. In the text, a story is described, but there is no direct quotation showing exactly what was discussed or how the story incorporates quotations.

The student should have added in direct quotations, but he failed to do so at crucial points in the essay. Not addressing the direct quotations makes the work look sloppy and ill-informed. Overall the failure to include examples from the text dropped the grade of the essay immensely.

When analyzing the rhetoric in a passage, it is best to find direct quotations to prove the strategies you are pointing out. Do not mention a strategy if you cannot show how the author used it in the text; otherwise, it seems like you are making it up.

Don’t Add Fluff

Students that have no more to write should stop writing. Adding in useless fluff to pad your essay and make it seem longer will only hurt the overall grade. Here is what some fluff looks like in the second essay:

AP® English Language FRQs

The student’s sentence adds nothing to the discussion of Louv’s rhetorical strategies. The sentence “…writes a striking piece on the separation between people and nature…” is almost entirely superfluous. There are many other, better ways to explain and introduce the topic.

It is better to write nothing or a very short sentence then to add unnecessary and redundant sentences to your essay. As you are working, make sure that every word counts; you don’t have enough time to spare to write sentences that add nothing to your overall argument.

AP® Readers’ Tips

  • Pay attention to both the holistic (overall) and analytic (particular) views of the piece. You will need to understand both the text as a whole and the specific parts of the text to analyze it effectively.
  • Don’t just analyze the rhetoric used, but instead connect the rhetoric to the specific purpose of the author. This rule applies to any rhetorical analysis essay.

The Third Essay (Argument)

The third and last essay of the FRQ does not respond to a particular text. Instead, the prompt focuses on crafting an argument about a particular issue. Your essay will need to argue a particular position, though most of the questions put forth by the exam will not be simple either/or questions.

Let’s look at the prompt for the third essay from 2013:

AP® English Language FRQs

Before we get into the do’s and don’ts of the essay, let’s talk about the particular challenge of this task. This particular task would be tough to write about because it deals with the philosophical ideas around the concept of ownership.

Be advised that you need to read the third prompt carefully. It is easy to fall into the trap of writing off-prompt because you misread or do not quite understand what it is asking you to do. In this case, you are asked to explain the relationship between the concept of ownership and sense of self, so an essay that doesn’t address that relationship would be off-task.

Always read the prompt carefully.

A few of the most important things you can do to ensure you score well on the essay include providing strong examples and crafting a strong thesis.

Provide Strong Examples to Substantiate Your Reasoning

There is always a need when arguing to provide strong examples to make your reasons and argument clear. In the student writing below they go to great lengths to provide strong examples of their argument:

AP® English Language FRQs

The student writes a very thorough explanation of the concept of ownership – explaining how there are varying degrees or definitions of ownership. There can be the ownership of a thing or object, and then there is the ownership of thoughts and ideas.

The student then uses the example of Candide to show how ownership of thoughts or ideas is related to the sense of self –“I’ve added the experience and memory of reading it to my personal concept of myself and my story”. The way that this student connects the concept of ownership to the concept of self is both clear and tangible – leading to a high score.

As you write, be sure to include strong examples that are clearly explained to the reader. The more clarity you have when writing, the easier it will be to understand what you are saying – and the stronger your argument and reasoning will be in the end.

Have a Strong Thesis and a Clear Argument

An excellent essay will have a strong thesis – usually provided in the first paragraph of the essay – that clearly expresses the argument of the author. A strong thesis in a timed essay like those on the AP® English Language exam should clearly articulate the claim.

Let’s take a look at one example of how one student articulated her thesis:

AP® English Language FRQs

The student writes a very strong introduction, but the best part of her intro is the clarity of her thesis. She goes into detail about what it means to own something, defining the concept. She then attaches that to her claim that, “The verb ‘to own’ doesn’t just mean to have something, it means we know something, or that we have made it a part of ourselves”.

The connection between her central argument (that owning things makes them a part of you) and the rest of her essay is established clearly through the rest of her essay. She provides three valid reasons that are fleshed out through each of the paragraphs, and each of those reasons works to establish the claim and thesis she created in the introduction.

As you write, be sure that your thesis is clear. Don’t muddle your writing by failing to establish a strong claim or craft a clear thesis.

If we take a look at the essay samples from 2013, there are few examples that stand out as don’ts. In particular, you should avoid these things.

Don’t Go Off Topic

One of the cardinal sins of essay writing is to go off-topic. Students that fail to address the prompt are sure to get a very low score.

Let’s take a look at a sample from an essay that goes off-topic and fails to adequately address the prompt:

AP® English Language FRQs

The student does not address the prompt properly in their essay. The student isn’t able to explain clearly what he means by ownership, and while he tries to make a connection between ownership and the sense of self, a strong link is never established.

The most shining example of how this student is seemingly off-task is in his short explanation of how ownership affects character. He says that “ownership is detriment to the person’s objects, it crafts the person’s character” but then goes into no detail about what that means. He then goes on to talk about the sense of self coming from protecting valuables.

This essay dances around the topic but it never quite makes a connection or any sense. Do not make the same mistake. Be sure of what the essay is asking you to write, and always keep on task – working to answer the prompt.

Don’t Give Simple Explanations for the Evidence Used

If you provide evidence from your thinking or the prompt, be sure that you explain it well. A simple explanation for the evidence you use is indicative of poor reasoning or sloppy work. Let’s examine the paragraph below:

AP® English Language FRQs

The student doesn’t seem quite to understand exactly what Aristotle means in his quote. The student points to not coveting the property of someone else as development of “moral character” but doesn’t go on to explain how it is morality and not merely satisfaction that causes a person not to steal or covet.

The student doesn’t elaborate on how property instills moral character, and he seems not to be able to stick to a single idea of how it teaches things like “responsibility” or “how to be fair” – he simply places these ideas in his writing and hopes that they will stand for themselves. They don’t.

In your writing, you must not only understand what you are using as evidence, but you must also be sure to explain yourself clearly. The essay above could have been spectacular if the student had explained how physical ownership translates to moral development in a clear and logical fashion.

  • Keep track of all parts of the prompt. One of the easiest ways to drop points is to forget to answer an important aspect of the prompt. In the case of the 2013 prompt, the essay needs to discuss the relationship between ownership and sense of self.
  • Try to reference literary examples in your writing. There wasn’t much opportunity to reference readings in the 2013 prompt, but if you can reference the different literature you have read as evidence, it can help boost your scores.

General AP® Readers’ Tips

Make a plan. One of the best things you can do for any essay you are writing under a time crunch is to create a thought-out strategy. Sometimes, in the heat of writing, it is easy to forget where we are in our arguments. Having a simple outline can save you from that misfortune.

Answer the question in your introduction, and be direct. This is one of the easiest ways to ensure you get a higher score.

Clearly indent your paragraphs, and ensure that you always have an easy-to-navigate structure. Topic sentences are a must, so make sure those figure into your structure.

Use evidence especially quotes from the texts, and explain what they mean. You need to make an explicit connection between the evidence you use, and how it supports your points.

Part of all great writing is variety. Vary your sentence structures; don’t make all of your sentences short or choppy, but instead try to inject some creativity into your writing. Utilize transitions, complex sentences, and elevated diction in your writing.

Use active voice, and make every word add to the paper as a whole. Avoid fluff; you don’t want your work to look bad because you are trying to pad your word count.

Go Forth and Conquer

Now that you better understand the expectations of the AP® Language and Composition FRQ section, you are one step closer to getting your five on the exam. Take what you have learned in this guide, and work on applying it to your writing. So, now it is time to go practice to perfection.

If you have any more tips or awesome ideas for how to study for the AP® English Language FRQ add them in the comments below.

Looking for AP® English Language practice?

Kickstart your AP® English Language prep with Albert. Start your AP® exam prep today .

Interested in a school license?​

Popular posts.

AP® Physics I score calculator

AP® Score Calculators

Simulate how different MCQ and FRQ scores translate into AP® scores

ap synthesis essay 2013

AP® Review Guides

The ultimate review guides for AP® subjects to help you plan and structure your prep.

ap synthesis essay 2013

Core Subject Review Guides

Review the most important topics in Physics and Algebra 1 .

ap synthesis essay 2013

SAT® Score Calculator

See how scores on each section impacts your overall SAT® score

ap synthesis essay 2013

ACT® Score Calculator

See how scores on each section impacts your overall ACT® score

ap synthesis essay 2013

Grammar Review Hub

Comprehensive review of grammar skills

ap synthesis essay 2013

AP® Posters

Download updated posters summarizing the main topics and structure for each AP® exam.

IMAGES

  1. AmStud

    ap synthesis essay 2013

  2. How to Write an Introductory Paragraph for a Synthesis Essay

    ap synthesis essay 2013

  3. Ap Synthesis Essay Template

    ap synthesis essay 2013

  4. Ap synthesis essay rubric by Brooks Denise

    ap synthesis essay 2013

  5. How To Write A Good Ap English Synthesis Essay

    ap synthesis essay 2013

  6. synthesis essay on ap lang exam

    ap synthesis essay 2013

VIDEO

  1. How To End Procrastination Forever in 7 days

  2. Macrolides/Clinical uses & Bacterial targets/By Dr.Hamza Lectures

  3. Toward effective electrocatalytic C–N coupling for the synthesis of organic nitrogeno...

  4. Several temperature-sensitive mutant strains of E: coli display various characteristics Below are o…

  5. Writing Your Synthesis Essay

  6. Synthesis essay PROCESS

COMMENTS

  1. Synthesis Essay Materials

    The two synthesis essay questions below are examples of the question type that has been one of the three free-response questions on the AP English Language and Composition Exam as of the May 2007 exam. The synthesis question asks students to synthesize information from a variety of sources to inform their own discussion of a topic. Students are given a 15-minute reading period to accommodate ...

  2. AP English Language and Composition Past Exam Questions

    Download free-response questions from past exams along with scoring guidelines, sample responses from exam takers, and scoring distributions. If you are using assistive technology and need help accessing these PDFs in another format, contact Services for Students with Disabilities at 212-713-8333 or by email at [email protected].

  3. How to Write a Perfect Synthesis Essay for the AP Language Exam

    Paragraph 1: The prompt presents and briefly explains the topic that you'll be writing your synthesis essay about. That topic is the concept of eminent domain. Paragraph 2: The prompt presents a specific claim about the concept of eminent domain in this paragraph: Eminent domain is productive and beneficial.This paragraph instructs you to decide whether you want to defend, challenge, or ...

  4. How to Write the AP Lang Synthesis Essay + Example

    The AP Lang synthesis essay is the first of three essays included in the Free Response section of the AP Lang exam. The exam presents 6-7 sources that are organized around a specific topic, with two of those sources purely visual, including a single quantitative source (like a graph or pie chart). The remaining 4-5 sources are text-based ...

  5. How to Write the AP Lang Synthesis Essay with Example

    Synthesis Essay AP Lang Examples - Sample Answer. The following is an essay response I crafted to the above question. After reading the sample, I will break down what it does well and what areas can be improved. A STEM-focused curriculum is not as essential to providing a meaningful K-12 education. Because the majority of high school students ...

  6. PDF AP English Language and Composition 2013 Free-Response Questions

    organized essay that synthesizes at least three of the sources for support, examine the factors a group or agency should consider in memorializing an event or person and in creating a monument. Make sure your argument is central; use the sources to illustrate and support your reasoning. Avoid merely ... 2013 AP ® ENGLISH LANGUAGE ...

  7. PDF AP® ENGLISH LANGUAGE AND COMPOSITION

    8 - Effective. Essays earning a score of 8 effectively examine the factors a group or agency should consider in memorializing an event or person and in creating a monument. They develop their argument by effectively synthesizing* at least three of the sources. The evidence and explanations used are appropriate and convincing.

  8. PDF ENGLISH LANGUAGE AND COMPOSITION

    2013 AP ® ENGLISH LANGUAGE ... organized essay that synthesizes at least three of the sources for support, examine the factors a group or agency should consider in memorializing an event or person and in creating a monument. Make sure your argument is central; use the sources to illustrate and support your reasoning. Avoid merely

  9. Acing the AP® English Language and Composition Synthesis Essay

    The newest section of the AP® English Language and Composition Exam, the synthesis essay, is one of three essays you will be completing during the examination's 2-hour free-response period. However, you'll also have a 15-minute reading and planning period just for this essay, and if you use this time to plan effectively, you can't go wrong.

  10. PDF AP English Language and Composition Synthesis Essay Sample Student

    AP®English Language and Composition Synthesis Essay Sample Student Responses. The College Board: Connecting Students to College Success. The College Board is a not-for-profit membership association whose mission is to connect students to college success and opportunity. Founded in 1900, the association is composed of more than 5,000 schools ...

  11. AP Lang

    Synthesis question. : A synthesis question is a type of essay prompt that requires students to combine information from multiple sources to develop a cohesive argument or analysis. It tests the student's ability to synthesize information and draw connections between different ideas. Time management.

  12. PDF ENGLISH LANGUAGE AND COMPOSITION

    2013 AP ® ENGLISH LANGUAGE ... organized essay that synthesizes at least three of the sources for support, examine the factors a group or agency should consider in memorializing an event or person and in creating a monument. Make sure your argument is central; use the sources to illustrate and support your reasoning. Avoid merely

  13. How to tackle the synthesis essay in AP Lang?

    Here's a step-by-step guide to help you tackle the synthesis essay in AP Lang: 1. Read the prompt carefully: Before diving into the sources, make sure you fully understand the prompt and what it's asking. Identify the main idea, the perspective or argument you need to present, and consider any constraints or specific instructions mentioned in ...

  14. PDF AP® English Language

    AP English Language Scoring Rubric, Free-Response Question 1-3 | SG 1 Scoring Rubric for Question 1: Synthesis Essay 6 points Reporting Category Scoring Criteria Row A Thesis (0-1 points) 4.B 0 points For any of the following: • There is no defensible thesis. • The intended thesis only restates the prompt.

  15. AP English Language Exam Practice: Synthesis Study Plan

    Resources you need to improve your Synthesis essay on the AP English Language and Composition exam. Includes revelant readings and practice problems. Note: For best results, click to highlight and copy/paste this list into your Fiveable Rooms Task Card to automatically create individual tasks. Jumpstart your studying in 5 seconds!

  16. Writing a synthesis essay for AP Lang

    Sure, a synthesis essay is a type of essay that requires you to use multiple sources to create an argument. In an AP Lang synthesis essay, you'll typically be provided with the sources and will need to analyze them, identify the main ideas, and then connect those ideas to your central argument or thesis. Here's a step-by-step breakdown of how to write a strong synthesis essay for AP Lang: 1.

  17. PDF AP® ENGLISH LANGUAGE AND COMPOSITION

    Question 1. The essay's score should reflect the essay's quality as a whole. Remember that students had only 15 minutes to read the sources and 40 minutes to write; the essay, therefore, is not a finished product and should not be judged by standards appropriate for an out-of-class assignment. Evaluate the essay as a draft, making certain ...

  18. AP Synthesis Essay Examples?

    Hi there! It's always a good idea to look for examples to better understand the format and expectations of an AP synthesis essay. I suggest you check out the following resources: 1. College Board: The College Board website is an excellent source as they create and administer the AP exams. You can find examples of synthesis essays from their past prompts, student responses, and scoring guidelines.

  19. Short Essay: AP Language Synthesis

    Remember, in an actual AP Synthesis Essay, you would need to reference the sources more directly, including specific evidence and quotations, and provide in-text citations (Source A, Source B, etc.) within the body of your essay. ... In 2013, he graduated from Edinburgh Napier University with a BEng(Hons) in Computing, with a focus on social ...

  20. The Ultimate Guide to 2013 AP® English Language FRQs

    On the 2013 FRQ section of the AP® exam, the synthesis essay focuses on the planning and consideration that goes into building a monument. The complete prompt for the section is below: If we break down the task, it is asking you to use the six sources to create a "coherent, well-developed argument" from your position about the factors a ...

  21. 2013 AP Lang Exam Synthesis Essay: Monuments

    2013 AP Lang Exam Synthesis Essay: Monuments. The need to memorialize events or people is complex; in some cases, mo... View more. Subject. AP English Language & Composition. 999+ Documents. Students shared 1335 documents in this course. Level AP. School iLEAD Online - Acton-CA. Academic year: 2022/2023.

  22. PDF AP English Language and Composition

    AP ® English Language ... Synthesis Essay 6 points . Urban rewilding is an effort to restore natural ecological processes and habitats in city environments. Many cities around th e world have embraced rewilding as part of larger movements to promote ecological conservation and environmentally friendly design. Now, a movement to promote urban

  23. PDF AP English Language and Composition

    AP ® English Language ... Synthesis Essay 6 points . Vertical farms are indoor agricultural facilities in which plants are grown, often in a hydroponic ( soilless) environment, on tall stacks of shelves. Plants are given water, nutrients, and light mostly through automated processes. Advocates say that vertical farms are key to providing food ...